Pediatric GU Flashcards

1
Q
  1. A female newborn is found to have a sacral mass. On MR this is lobulated with well-defined margins and heterogeneous SI on T1 with high, intermediate and low SI areas. It is a predominantly eternal lesion, covered by skin with only minimal presacral component. Which is the most likely diagnosis?

A. Myelomeningocele

B. Sacrococcygeal teratoma

C. Rectal duplication cysts

D. Haemangioma

E. Lymphoma

A

B. Sacrococcygeal teratoma

Fat, soft tissue and calcification cause the heterogeneous SI on T1. This is the most common solid tumour in the
newborn, more common in females.

How well did you know this?
1
Not at all
2
3
4
5
Perfectly
2
Q

@# (CNS) 11. Which is the most prevalent intraventricular tumour?

A. Meningioma

B. Choroid plexus paplilloma

C. Astrocytoma

D. Dermoid

E. Subependymoma

A

C. Astrocytoma

Ependymomas followed by astrocytoma are the most common.

How well did you know this?
1
Not at all
2
3
4
5
Perfectly
3
Q
  1. Regarding neuroblastomas:

A. Produces catecholamines in all cases

B. Are most common in 5-10 years

C. Have a better prognosis in a younger child

D. The most common site following the adrenals is the neck

E. Esthesioneuroblastoma is a very malignant tumour arising from olfactory mucosa

A

E. Esthesioneuroblastoma is a very malignant tumour arising from olfactory mucosa

Neuroblastomas produce catecholamines in 95% of cases.

The peak age of diagnosis is < 3 and there is a better prognosis in older children.

The most common site is adrenals, followed by chest, followed by neck.

How well did you know this?
1
Not at all
2
3
4
5
Perfectly
4
Q
  1. An 8-month-old has a neuroblastoma in the right adrenal with metastatic spread to the liver, bone marrow and skin. What stage of disease does he have?

A. Stage 2B

B. 3

C. 2A

D. 4

E. 4S

A

E. 4S

Stage 4S applies to infants who demonstrate a tumor on one side of the body, as seen in Stages 1 and 2, but also have affected liver, skin, and/or bone marrow.

Stage 4 involves distant lymph nodes and bone with or without bone marrow/ liver/other organs.

How well did you know this?
1
Not at all
2
3
4
5
Perfectly
5
Q
  1. Currarinos triad is a rare association of anomalies including anorectal malformations, lumbrosacral anomalies and which of the following?

A. Meconium ileus

B. Presacral mass

C. Ileal atresia

D. Hirshsprungs

E. Cloacal extrophy

A

B. Presacral mass

This may be teratoma, anterior meningocele or enteric cyst.

How well did you know this?
1
Not at all
2
3
4
5
Perfectly
6
Q
  1. Regarding imaging of duplicated kidneys:

A. Micturating Cystourethroram (MCUG) shows reflux into the upper pole

B. Duplicated kidneys tend to be smaller

C. Duplex kidneys may have more than 1 renal artery and vein

D. US shows dilatation of the lower pole calyx with a normal upper pole

E. A band of medulla crosses the cortex of the kidney onUS

A

C. Duplex kidneys may have more than 1 renal artery and vein

In cases of duplicated kidneys, MCUG shows reflux into the lower pole.

Duplicated kidneys tend to be larger and US shows dilatation of the upper pole calyx and a normal lower pole.

A band of cortex can be seen to cross the medulla.

How well did you know this?
1
Not at all
2
3
4
5
Perfectly
7
Q
  1. Regarding posterior urethral valves:

A. Are seen equally on males and females

B. Associated with antenatal polyhydramnios

C. Gradual change in calibre occurs in the posterior urethra

D. Are associated with a smooth walled urinary bladder

E. Are associated with vesicoureteric reflux

A

E. Are associated with vesicoureteric reflux

Occurs in boys. Associated with oligohydramnios, an abrupt change in calibre of the posterior urethra and a
trabeculated urinary bladder wall.

How well did you know this?
1
Not at all
2
3
4
5
Perfectly
8
Q
  1. Neonatal ultrasound shows kidneys whch are enlarged bilaterally hyperechoic with loss of normal corticomedullary differentiation. Cysts are noted which are less than 1cm. There are focal rosettes of radially oriented
    dilated collecting tubules. What is the diagnosis?

A. Autosomal Recessive Polycistic Kidney Disease (ARPKD)

B. Autosomal Dominant Polycistic Kidney Disease (ADPKD)

C. Asymmetrical large renal cysts

D. Tuberous sclerosis

E. Multicystic Dysplastic Kidney (MCDK)

A

A. Autosomal Recessive Polycistic Kidney Disease (ARPKD)

ADPKD have normal renal echogenicity. Small cyst may not be immediately identified on ultrasound.

How well did you know this?
1
Not at all
2
3
4
5
Perfectly
9
Q

@# 46. Which of the following are common features in mesoblastic nephroma?

A. Cystic areas of necrosis

B. Areas of haemorrhage

C. Calcification

D. Invasion of vessels

E. Polyhydramnios in pregnancy

A

E. Polyhydramnios in pregnancy

Cystic areas of necrosis, areas of haemorrhage and calcification are uncommon.

How well did you know this?
1
Not at all
2
3
4
5
Perfectly
10
Q

@# 47. Nephroblastomatosis, Beckwith-Wiedemann syndrome, chromosome 11 abnormalities and trisomy 18 are all associations of which of the following?

A. Autosomal recessive polycystic kidney disease

B. Neuroblastoma

C. Nephroblastomatosis

D. Wilms’ tumour

E. Multilocular cystic nephroma

A

D. Wilms’ tumour

These are all known associations of Wilm’s tumor

How well did you know this?
1
Not at all
2
3
4
5
Perfectly
11
Q
  1. A female infant is found to have a pelvic mass. She is systemically well. US shows a well-defined fluid-filled
    cavity between the bladder and rectum with some debris but no increased vascularity. There is uterine distension
    and echogenic debris within the vagina. Hydronephrosis is noted. Which is the diagnosis?

A. Pelvic abscess

B. Hydrometocolpos

C. Ovarian tumour

D. Fallopian tube torsion

E. Sacrococcygeal teratoma

A

B. Hydrometocolpos

MR shows mixed SI on T1 & T2 due to blood products and may be associated with uterine and cervical anomalies.

How well did you know this?
1
Not at all
2
3
4
5
Perfectly
12
Q
  1. Which of the following is a feature of neuroblastoma?

A. Large hypoechoic mass arising the suprarenal region

B. Calcification in most cases on CT

C. No increased vascularity

D. Echogenic liver lesions

E. Vascular invasion

A

A. Large hypoechoic mass arising the suprarenal region

Low attenuation metastatic liver lesion are echogenic on US. May have lytic/sclerotic or mixed bone metastases,
increased vascularity and vascular encasement.

How well did you know this?
1
Not at all
2
3
4
5
Perfectly
13
Q

@# 50. A child with Wilms’ tumour has exomphalos, macroglossia, gigantism and hepatomegaly. Which is the diagnosis?

A. Hemihypertrophy

B. Drash syndrome

C. Beckwith-Wiedemann

D. Trisomy 18

E. Trisomy 21

A

C. Beckwith-Wiedemann

A, B and C are all associations of Wilm’s tumour. 10-20% of patients with Beckwith-Wiedemann develop
Wilm’s tumour.

How well did you know this?
1
Not at all
2
3
4
5
Perfectly
14
Q

4) A 3-year-old child with aniridia is found to have a palpable abdominal mass. The mass is shown to arise from
a kidney and to contain cystic elements on ultrasound scan. No calcification is seen in the tumour on CT. Which
of the following diagnoses is the most likely?

a. renal cell carcinoma

b. neuroblastoma

c. angiomyolipoma

d. Wilms’ tumour

e. ossifying renal tumour of infancy

A

d. Wilms’ tumour

Wilms’ tumour is the commonest renal tumour of childhood. Seventyfive per cent occur in children under 5 years,
5–10% are bilateral and 10% are multifocal. Calcification is seen in less than 15%. Nephroblastomatosis is a
precursor, and the disease is associated with the WT1 and WT2 genes of chromosome 11. The WT1 abnormal
gene is found in the WAGR syndrome of Wilms’ tumour, aniridia, genitourinary abnormalities and learning
disability. It is also found in the DRASH syndrome of male pseudohermaphroditism and progressive
glomerulonephritis. The abnormal WT2 gene is found with the Beckwith–Wiedemann syndrome and
hemihypertrophy.

How well did you know this?
1
Not at all
2
3
4
5
Perfectly
15
Q

9) A baby boy is investigated for renal failure. The imaging findings are of bladder distension and a posterior
urethral valve, ureters measuring 10 mm in diameter bilaterally, undescended testes and widely separated
abdominal rectus muscles. What is the most likely diagnosis?

a. prune-belly syndrome

b. primary vesicoureteric reflux

c. developmental aperistalsis of the distal ureter

d. neuropathic bladder

e. ureterocele

A

a. prune-belly syndrome

(a) to (e) are all causes of a megaureter – that is, a ureter over 7 mm in diameter. The hallmark of prune-belly
syndrome is a distended bladder, and it is associated with posterior urethral valves. Bladder distension causes the
triad of widely spaced abdominal rectus muscles, hydroureteronephrosis and cryptorchidism. The last occurs
because the large bladder interferes with testicular descent.

How well did you know this?
1
Not at all
2
3
4
5
Perfectly
16
Q

24) Abdominal ultrasound is performed on a neonate on the highdependency unit to investigate a palpable mass.
A heterogeneous avascular suprarenal mass is identified. Cystic change and a peripheral hyperechoic rim develop
over a series of scans. Which of the following is the most likely cause of the abdominal mass?

a. nephroblastoma

b. neuroblastoma

c. adrenal haematoma

d. phaeochromocytoma

e. myolipoma

A

c. adrenal haematoma

Adrenal haemorrhage is not only the commonest cause of neonatal adrenal mass, but is also more likely to be seen in neonates in a high-dependency unit because it is associated with perinatal stress, hypoxia, septicaemia and hypotension. It can be unilateral or bilateral, but, even when bilateral, it does not usually cause adrenal insufficiency. Initially, the haematoma appears as an avascular heterogeneous mass on ultrasound scan that becomes cystic and smaller over a period of weeks.
A hyperechoic rim can form, representing peripheral calcification. Haematomas can become infected, resulting
in an abscess.

Neuroblastoma is the main differential diagnosis; on ultrasound scan, it appears as a hyperechoic
mass that can have internal flecks of calcification. Repeat ultrasound scan at 1 week will not show the changes that haematoma undergoes.

How well did you know this?
1
Not at all
2
3
4
5
Perfectly
17
Q

28) A 5-year-old boy has recurrent urinary tract infection. A micturating cystourethrogram is performed, during
which contrast from the bladder enters both ureters and reaches the pelvis and calyces bilaterally without any
urinary tract dilatation. Of what grade is this vesicoureteric reflux?

a. I

b. II

c. III

d. IV

e. V

A

b. II

The international grading system for vesicoureteric reflux divides reflux into five classes. Grade I describes reflux
into the ureter only, with grade II referring to reflux into the pelvicalyceal system without calyceal dilatation or
blunting. Grade III reflux is associated with mild pelvicalyceal and ureteric dilatation, though forniceal angles
remain distinct. Grade IV is associated with a tortuous ureter and moderate dilatation of the pelvicalyceal system,
with blunting of the forniceal angles. Grade V reflux describes grossly dilated tortuous ureters with marked
pelvicalyceal dilatation and obliteration of the forniceal angles.

How well did you know this?
1
Not at all
2
3
4
5
Perfectly
18
Q

36) A 1-month-old girl has liquid discharge from the umbilicus. Which of the following provides a suitable
explanation?

a. vesicourachal diverticulum

b. urachal cyst

c. patent urachus

d. bladder exstrophy–epispadias complex

e. cloacal exstrophy

A

c. patent urachus

Embryologically, the cloaca is divided by the urorectal septum into a dorsal part that develops into the rectum
and a ventral part that gives rise to the allantois, bladder and urogenital sinus. The wolffian and mu¨llerian ducts
drain into the ventral cloaca. The allantois becomes the urachus, which is the umbilical attachment of the bladder.
Ordinarily, this atrophies to become the umbilical ligament. If it remains patent throughout its entire length, urine
can drain via the umbilicus. A urachal sinus and a vesicourachal diverticulum describe patent portions of the
urachus at the umbilical and bladder ends respectively.

How well did you know this?
1
Not at all
2
3
4
5
Perfectly
19
Q

34) Ultrasound scan of the abdomen of a newborn girl reveals an abdominopelvic cyst. It is thin walled and
anechoic, and has a ‘daughter cyst’. Of the following which is the most likely diagnosis?

a. Wilms’ tumour

b. ovarian cyst

c. cystic lymphatic malformation

d. choledochal cyst

e. cystic teratoma

A

b. ovarian cyst

Ovarian cysts in the newborn are more common than enteric duplication cysts, giant meconium pseudocysts,cystic lymphatic malformations or choledochal cysts. Other rarer causes of intraabdominal cystic structures in the newborn include cystic teratomas, gastric teratomas, cystic granulosa cell tumour of the ovary, ovarian teratomas and cystadenomas. Ovarian cysts may become echogenic due to the haemorrhage that can occur if they tort (twist).
These cysts also have associated normal ovarian tissue, and the daughter cyst represents a follicle along the wall.

Wilms’ tumours are solid and occur later in life.

A giant meconium pseudocyst has a thick echogenic wall, and viscous echogenic contents. It is formed by meconium leak following fetal bowel perforation due to intestinal obstruction in meconium ileus, ileal atresia or volvulus. Twenty-five per cent show peritoneal or cyst calcification,which is pathognomonic for meconium pseudocyst. Bowel obstruction may be present also.

Cystic lymphatic malformations appear as large, well-circumscribed, cystic, thin-walled structures with multiple thin septa. Internally, the fluid can be echo free or echoic because of haemorrhage, debris, chyle or infection. Mesenteric,omental or retroperitoneal cysts are seen.

Choledochal cysts are subhepatic or in the porta hepatis. They are seen separate from the gallbladder and are round, tubular or teardrop shaped, and connected to the biliary tree.

How well did you know this?
1
Not at all
2
3
4
5
Perfectly
20
Q

41) At a 20-week, antenatal ultrasound scan, a fetus has a renal pelvis diameter measured at 7 mm unilaterally.
When the scan is repeated at 35 weeks, the renal pelvis measures 12 mm. Which of the following is the most
appropriate follow-up for the neonate?

a. discharge

b. ultrasound scan at 6 weeks

c. ultrasound scan within 24 hours of birth

d. micturating cystourethrogram soon after birth

e. 99mTc-labelled DMSA scan soon after birth

A

c. ultrasound scan within 24 hours of birth

Pyelectasis can be regarded as a renal pelvis diameter greater than 4– 5 mm at 20 weeks’ gestation or 7 mm at 33
weeks, or above 10 mm at birth. Local protocol varies as to which fetuses to follow up and how. Some centres
perform an ultrasound scan on all neonates who have had a renal pelvis diameter of above 5 mm at any point;
others may only investigate if there is a renal pelvis above 10 mm persisting to birth. Ultrasound scan soon after
birth, however, is the best way of detecting severe obstructive pathology, such as posterior urethral valves, that
may warrant rapid surgical intervention. On this scan, if there is persisting dilatation above 10 mm, antibiotic
prophylaxis and micturating cystourethrogram (MCUG) are appropriate. If reflux is seen on the MCUG, DMSA
and repeat ultrasound scan at 6 weeks are appropriate. DMSA is used to assess parenchyma for scarring. If no
reflux is seen, MAG3 scan within 6 weeks is suggested to look for obstruction. If the renal pelvis diameter is less
than 10 mm within 24 hours of birth, followup ultrasound scan 6 weeks later is suggested.

How well did you know this?
1
Not at all
2
3
4
5
Perfectly
21
Q

54) A 12-year-old boy is investigated for abdominal pain. On ultrasound scan, there are large, echo-free, septated,
cystic collections around both kidneys causing scalloping of the renal outline. On CT, these collections are close
to water density. Which of the following is the most likely diagnosis?

a. renal lymphangiectasia

b. bilateral Wilms’ tumours

c. bilateral adrenal neuroblastoma

d. bilateral hydronephrosis

e. medullary sponge kidney

A

a. renal lymphangiectasia

Renal lymphangiectasia is a very rare developmental malformation probably caused by a failure of the developing
kidney lymphatics to establish communication with the extrarenal lymphatic system. Abnormal lymphatic
channels dilate, resulting in cystic lesions in the parapelvic, perinephric and, less commonly, retroperitoneal
regions. The lesions are of water attenuate

How well did you know this?
1
Not at all
2
3
4
5
Perfectly
22
Q

@# 55) A 15-year-old female, whose father had progressive renal failure, presents with anaemia, polyuria and
haematuria. On ultrasound scan, her kidneys are small and smooth. Which associated finding is most likely?

a. pancreatic cysts

b. posterior fossa haemangioblastoma

c. cystocele

d. nerve deafness

e. hypertension

A

d. nerve deafness

Alport’s syndrome or chronic hereditary nephritis is the unifying diagnosis. It is inherited, probably in an
autosomal dominant fashion. Ocular abnormalities can also occur, including congenital cataracts, nystagmus, myopia and spherophakia. Hypertension is not a feature. The renal impairment is progressive in affected males
but non-progressive in females.

Cerebellar and retinal haemangioblastomas occur in von Hippel–Lindau syndrome, along with renal, pancreatic and adrenal cysts.

How well did you know this?
1
Not at all
2
3
4
5
Perfectly
23
Q

75) Which of the following features on ultrasound scan is most suggestive of a horseshoe kidney?

a. unilateral upper pole calyx dilatation

b. unilateral lower pole calyx dilatation

c. bilateral upper pole calyx dilatation

d. laterally oriented renal long axis

e. medially oriented renal long axis

A

e. medially oriented renal long axis

Ninety per cent of horseshoe kidneys are joined at the lower poles by a parenchymal or fibrous isthmus. The
isthmus lies at the L4–5 level, where renal ascent is arrested by the inferior mesenteric artery. The pelves and
ureters are anterior, and pelviureteric junction obstruction is more common. Incidence is 1–4/1000 births, making
it the commonest renal fusion abnormality. Cardiovascular, skeletal, central nervous system, anorectal and
genitourinary malformations are associated. There are associations with trisomy 18 and Turner’s syndrome.
Associated genitourinary anomalies include hypospadias, undescended testes, bicornuate uterus and ureteral
duplication. The incidence of infection and stones increases in a horseshoe kidney.

How well did you know this?
1
Not at all
2
3
4
5
Perfectly
24
Q

(GU) 77) On an antenatal ultrasound scan, unilateral fetal ureteric dilatation is identified. Follow-up imaging
after birth reveals persistent dilatation of a non-tortuous ureter that is aperistaltic in its distal segment. There are
no other associated urogenital anomalies. Which of the following is the most appropriate diagnosis?

a. prune-belly syndrome

b. primary vesicoureteric reflux

c. primary megaureter

d. neuropathic bladder

e. ureterocele

A

c. primary megaureter

The primary megaureter occurs because of developmental aperistalsis of the distal ureter. The ureter tends to be
dilated but straight in a congenital primary megaureter. Primary vesicoureteric reflux and obstructive secondary
megaureter tend to cause tortuous ureteric dilatation

How well did you know this?
1
Not at all
2
3
4
5
Perfectly
25
Q

79) On antenatal ultrasound scan, oligohydramnios, non-visualization of the urinary bladder and bilateral enlarged hyperechoic kidneys are recorded. Renal failure ensues within the first month of life. An older sibling aged 5 years has established portal hypertension. The parents are phenotypically normal. What are the odds of further
offspring of the same parents having the same disease?

a. zero

b. 1 in 5

c. 1 in 4

d. 1 in 2

e. unity

A

c. 1 in 4

Infantile polycystic kidney disease is autosomal recessive. Both parents and half the children will be carriers. The
sonographic hallmark is enlarged echogenic kidneys. The individual cysts are small, measuring 1–2 mm, and can
be defined only with a high-resolution probe. Autosomal kidney disease affects the liver also. The less severe the
renal disease, the more severe the hepatic periportal fibrosis.

How well did you know this?
1
Not at all
2
3
4
5
Perfectly
26
Q

(GU) 79) An IVU is performed on a teenager. On a 15-minute, full-length radiograph, the right ureter and
collecting system appear normal, while the left collecting system is displaced laterally and inferiorly, giving a
‘drooping flower’ appearance. What is the most likely congenital anomaly?

a. bilateral ureteral duplication

b. left ureteral duplication

c. left ureteral diverticulum

d. recently passed left ureteric stone

e. crossed fused ectopia

A

b. left ureteral duplication

The ‘drooping flower’ appearance on IVU occurs with a duplicated system when the obstructed dilated upper
pole collecting system displaces the contrast-opacified, lower collecting system laterally and inferiorly. The
Weigert–Meyer rule describes the commonest relationship of insertion of the duplicated ureters into the bladder.
The upper collecting system ureter inserts ectopically into the bladder inferior and medial to the orthotopic ureter,
which enters the bladder near the trigone. The ectopic ureter may be stenotic or obstructed with or without
ureterocele

How well did you know this?
1
Not at all
2
3
4
5
Perfectly
27
Q

87) Which of the following would be regarded as abnormal during ultrasound scan of the kidneys in a neonate?

a. echogenicity of the renal cortex similar to that of liver

b. large hyperechoic medullary pyramids

c. paucity of renal sinus fat

d. lobulated renal contour

e. echogenic septum at anterosuperior margin of the kidney

A

b. large hyperechoic medullary pyramids

The average length of the normal neonatal kidney is 4.5 cm. Ultrasound appearances of the kidney in the neonate
and infant show a number of differences from those of the older child and adult. Cortical echogenicity is increased,
as the glomeruli form 20% of the cortex in the neonate but only 9% in the adult, and may be comparable to that
of the liver or the spleen. The medullary pyramids are larger and more hypoechoic than in older children and
adults, and there may be little or no fat in the renal sinus. Persistent fetal lobulation may give the kidneys a
lobulated contour. In addition, echogenic septa may be normally seen at the anterosuperior or posteroinferior
margins of the kidney, representing the sites of fusion of metanephric elements. Most kidneys attain an adult
pattern by 6 months of age

How well did you know this?
1
Not at all
2
3
4
5
Perfectly
28
Q

96) A 16-year-old female with a history of imperforate anus corrected soon after birth is investigated for primary
amenorrhoea. Ultrasound scan shows the uterus and upper vagina distended by moderately echogenic material.
Which of the following is the most likely cause of the amenorrhoea?

a. longitudinal vaginal septum

b. transverse vaginal septum

c. cervical dysgenesis

d. obstructed uterine horn

e. endometritis

A

b. transverse vaginal septum

Haematometrocolpos is described, which at this age can be due to a transverse vaginal septum or imperforate
hymen. There is an association with imperforate anus, hydronephrosis, renal agenesis and dysplasia, polycystic
kidneys, duplication of the vagina and uterus, sacral hypoplasia and oesophageal atresia. Cervical dysgenesis and
obstructed uterine horn would produce haematometra.

How well did you know this?
1
Not at all
2
3
4
5
Perfectly
29
Q

3 An antenatal US performed at 20 weeks demonstrated a right renal pelvis with an AP diameter of 5 mm. A 32
week scan shows the diameter to be 10 mm. The post-natal US at 4 days confirms unilateral right-sided neonatal
hydronephrosis. What is the commonest cause of these findings?

(a) Ectopic ureterocoele

(b) Pelvi-ureteric junction obstruction

(c) Posterior urethral valve

(d) Prune-belly syndrome

(e) Vesico-ureteric junction obstruction

A

(b) Pelvi-ureteric junction obstruction

The most common causes of antenatal hydronephrosis (regardless of gender), in order of decreasing frequency
are: PUJ obstruction (22%), posterior urethral valve (18%), ectopic ureterocoele (14%), Prune-belly syndrome
(12%), and VUJ obstruction (8%)

How well did you know this?
1
Not at all
2
3
4
5
Perfectly
30
Q

6 A 3 year old girl presents with increasing abdominal girth. US demonstrates a mass within the region of the left
kidney. The patient proceeds to CT examination. Which of the following findings favours Wilms’ tumour over
neuroblastoma?

(a) Calcification within the mass

(b) Mass crosses the mid line

(c) Mass encases and lifts the aorta

(d) Mass invades into the IVC

(e) Presence of bone metastases

A

(d) Mass invades into the IVC

Classically neuroblastoma encases vessels, whereas Wilm’s tumours may invade vessels. Calcification is more
common in neuroblastoma (85-95%) than Wilm’s (15%), neuroblastoma commonly crosses the midline, only
10% of Wilm’s tumours are bilateral. Neuroblastoma typically metastasises to bone and lymph nodes; Wilm’s to
the liver, lungs and lymph nodes.

How well did you know this?
1
Not at all
2
3
4
5
Perfectly
31
Q

A 3 year old boy presents with abdominal swelling. US and subsequent CT examination show a large 12 cm mass
inferior to the liver. There is distortion of the renal parenchyma and apparent exophytic growth. There is poor
enhancement and the aorta and IVC are displaced. Which of the following features is not a known association?

(a) Aniridia

(b) Cerebellar ataxia

(c) Horseshoe kidney

(d) Macroglossia

(e) Male pseudohermaphroditism

A

(b) Cerebellar ataxia

Cerebellar ataxia is associated with neuroblastoma. Wilm’s is associated with WAGR (includes duplex/
horseshoe/ fused kidneys), Drash syndrome (male pseudo-hermaphroditism), and BeckwithWiedemann
syndrome (macroglossia). It is also associated with spontaneous aniridia and hemihypertrophy

How well did you know this?
1
Not at all
2
3
4
5
Perfectly
32
Q

9 A neonatal male patient failed to pass meconium by 48 hours and was found to have an imperforate anus of the
high malformation subtype. A colostomy was formed in the neonatal period and surgical repair completed in
infancy. At the age of 5 the patient presents with a UTI. What is the likely underlying aetiology of the UTI?

(a) Colovesical fistula

(b) Neurogenic bladder

(c) Pelvi-ureteric junction obstruction

(d) Rectourethral fistula

(e) Urethrocoele

A

(b) Neurogenic bladder

Anorectal malformation may be low (commonest), intermediate, or high, depending on the relation to the levator
sling. High malformations are associated with fistulas and a high association with GU (50%), and VACTERL
anomalies. History of surgical repair makes fistulas unlikely; post-op patients often suffer urinary incontinence;
the additional association of spinal anomalies make neurogenic bladder (which predisposes to UTI) a likelihood.
There is no association with PUJ obstruction or urethrocoele.

How well did you know this?
1
Not at all
2
3
4
5
Perfectly
33
Q

14 An 18 month old boy is found to have a neuroblastoma. A subsequent CT is performed for staging purposes.
This reveals the primary tumour and evidence of metastases in the skin, liver, lung and bone marrow.
What is the appropriate stage?

(a) Stage I

(b) Stage II

(c) Stage Ill

(d) Stage IV

(e) Stage IV-S

A

(d) Stage IV

Stage I limited to organ of origin.
Stage II regional spread, but does not cross midline.
Stage III tumour extends across midline, or is a unilateral tumour with contralateral LNs.
Stage IV: the tumour has spread to distant lymph nodes, bone, bone marrow, liver, skin or other organs.
Stage 1IV-S is a subset of neuroblastoma in infants < 1 year with a good prognosis

How well did you know this?
1
Not at all
2
3
4
5
Perfectly
34
Q

20 Which of the following conditions has the highest frequency of non-genitourinary anomalies?

(a) Ask-Upmark kidney

(b) Bilateral ectopia

(c) Crossed-fused ectopia

(d) Duplex kidney

(e) Horseshoe kidney

A

(e) Horseshoe kidney

Horseshoe kidneys are associated with other renal anomalies in 50%, and non-GU system anomalies in 30% (GI,
cardiac, skeletal). In 90% of crossed ectopia there will be fusion (crossed-fused ectopia); associated anomalies
are rare. Ask-Upmark kidney shows segmental hypoplasia; there is controversy over whether it is a congenital
anomaly or a sequelae of severe pyelonephritis. In the cited reference, no patients with duplex kidneys were found
to have non-GU related anomalies.

How well did you know this?
1
Not at all
2
3
4
5
Perfectly
35
Q

(GU) 24 A 10 year old boy, recently arrived from the Indian subcontinent, presents with vague abdominal
distension and discomfort. US shows bilateral multiple, non-communicating, well defined cystic lesions in the peripelvic region, renal parenchyma and in the perirenal spaces. CT shows the lesions to be homogeneous with no significant contrast enhancement. At MRI the lesions were of low signal intensity on T1W and high signal intensity on T2W, again without enhancement. Needle biopsy showed areas of connective tissue with an`endothelial lining. What is the likeliest diagnosis?

(a) Multicystic dysplastic kidney

(b) Multilocular cystic nephroma

(c) Tuberculosis

(d) Wilm’s tumour

(e) Renal lymphangiectasia

A

(e) Renal lymphangiectasia

Although rare, renal lymphangiectasia has a characteristic imaging appearance resulting from the failure of the
renal lymphatic tissue.

How well did you know this?
1
Not at all
2
3
4
5
Perfectly
36
Q

25 A 4 year old girl has a proven first UTI. E. coli is cultured and there is a good response to antibiotics. Her GP phones to ask your advice on the most appropriate way to proceed with imaging. What should you recommend?

(a) OMSA scan at 6 months

(b) MCUG at 6 weeks

(c) No imaging

(d) US acutely

(e) US at 6 weeks

A

(c) No imaging

Recent NICE guidelines recommend no imaging for simple (nonatypical) first UTls in infants 6 months - 3 years
and children over 3 years which respond to antibiotics within 48 hours. In children over 3 years with an atypical
UTI, an US should be urgently arranged, in those with recurrent UTls a 6 week US and 6 month DMSA scan
should be arranged; there is no role in this age group for MCUG. An US is always performed in those < 6 months
old (routinely at 6 wks), if this is abnormal, or there is an atypical organism, or recurrent UTls, MCUG is
performed. MCUG may be considered in those 6 months – 3 years if there is pelvic dilation on US, or a family
history of VUR

How well did you know this?
1
Not at all
2
3
4
5
Perfectly
37
Q

26 Which of he following types of polycystic kidney disease is associated with greatest degree of liver fibrosis?

(a) Autosomal dominant PKO

(b) Antenatal autosomal recessive PKO

(c) Infantile autosomal recessive PKO

(d) Juvenile autosomal recessive PKO

(e) Neonatal autosomal recessive PKO

A

(d) Juvenile autosomal recessive PKO

ARPKD is associated with liver fibrosis and portal hypertension. The degree of hepatic fibrosis is inversely related
to the degree of renal disease. The relative involvement is: antenatal (90%; renal disease; minimal hepatic
fibrosis), neonatal (60%; mild fibrosis) infantile (20%; moderate fibrosis) and juvenile (10%; gross liver fibrosis).
Juvenile type is postulated to be a primary hepatic fibrosis rather than an ARPKD.

How well did you know this?
1
Not at all
2
3
4
5
Perfectly
38
Q

33 A 30 year old man presents with recurrent loin pain. Ultrasound shows a horseshoe kidney. Which of the following statements is incorrect?

(a) The incidence at autopsy is 1:2,000

(b) Fusion of the kidneys occurs at the lower pole in 90%

(c) It is more common in males

(d) Ascent of the kidney is arrested at the L3 level by the inferior mesenteric artery

(e) It is seen in 60% of patients with Turners syndrome

A

(a) The incidence at autopsy is 1:2,000

It is the commonest fusion abnormality and is seen in 1:400 patients at autopsy. It can be complicated by
obstruction, reflux or stone formation. The isthmus can contain functioning renal tissue.

How well did you know this?
1
Not at all
2
3
4
5
Perfectly
39
Q

39 An antenatal US shows oligohydramnios, a protruberant abdomen, bilateral hydronephrosis and markedly dilated and tortuous ureter. The posterior urethra is dilated and there is a large distended thin-walled bladder. At birth the testicles are noted to be undescended. What is the most likely diagnosis?

(a) Cloacal abnormality

(b) Megacystis-microcolon-hypoperistalsis (MMH) syndrome

(c) Posterior urethral valves

(d) Prostatic rhabdomyosarcoma

(e) Prune-belly syndrome

A

(e) Prune-belly syndrome

Prune-belly syndrome is a triad of absent/ deficient abdominal muscules, renal dysplasia (bilateral hydroureters/
hydronephrosis), and cryptorchidism. Due to the associated absence of the prostate gland, the posterior urethra is
often dilated proximally - the entire urethra may be dilated. PUVs are a key differential, but the implied abdominal
wall defect, thin-walled bladder, and cryptorchidism go against this. MMH causes insufficiency of the abdominal
viscera, but occurs in females. Rhabdomyosarcoma would be unusual antenatally.

How well did you know this?
1
Not at all
2
3
4
5
Perfectly
40
Q

(GU) 42 A neonate is found to have an abdominal wall defect and bilateral cryptorchidism. Which of the following features would not support a syndromic diagnosis?

(a) Bilateral ureteromegaly

(b) Trabeculated low capacity bladder

(c) Widened prostatic urethra

(d) Urethral dilatation

(e) Asymmetric renal size

A

(b) Trabeculated low capacity bladder

The classical triad of prune-belly syndrome is abdominal wall defect/ weakness, bilateral cryptorchidism and
genito-urinary abnormalities. Typically, a thickened, non-trabeculated, large volume bladder is seen. Non GU
abnormalities affecting the lungs (CCAM, lung hypoplasia), musculoskeletal system (scoliosis, pectus
excavatum), cardiovascular (VSD, PDA, Fallots) and GI system (Hirschprung’s disease, malrotation) are also
seen

How well did you know this?
1
Not at all
2
3
4
5
Perfectly
41
Q

(GU) 46 A 3 year old undergoes CT of the abdomen which shows an 8 cm heterogeneous enhancing mass which does not extend across the midline, but which displaces major vessels. Which of the following conditions is not associated?

(a) Cryptorchidism

(b) Hypoplasia of the iris

(c) Hemihypertrophy

(d) Drash syndrome

(e) Proptosis

A

(e) Proptosis

The features described would suggest a diagnosis of Wilm’s tumour which has a number of associations including
Beckwith-Wiedemann syndrome, sporadic aniridia, hemihypertrophy and other genitourinary abnormalities
including Drash syndrome. Proptosis can be seen in neuroblastoma.

How well did you know this?
1
Not at all
2
3
4
5
Perfectly
42
Q

8 A 3 year old boy with a Wilm’s tumour undergoes CT examination to aid staging. The CT shows a large mass
arising from the right kidney, with liver lesions, vertebral lesions, and enlarged hilar lymph nodes. Based on these
findings, the stage is?

(a) Stage I

(b) Stage II

(c) Stage Ill

(d) Stage IV

(e) Stage V

A

(d) Stage IV

Stage I: tumour limited to the kidney.
Stage II: extension into the perinephric space +/-vessel invasion.
Stage III: LNs in the abdomen or pelvis, or peritoneal invasion.
Stage IV: haematogeneous spread to lung/ liver/ brain/ bone, or to LNs outside the abdomen or pelvis.
Stage V: bilateral renal tumors at diagnosis.

How well did you know this?
1
Not at all
2
3
4
5
Perfectly
43
Q

9 A neonate presents with a visible mass within the left flank. US confirms a solid intra-renal lesion. What is the
likeliest tumour?

(a) Mesoblastic nephroma

(b) Multi cystic dysplastic kidney

(c) Nephroblastomatosis

(d) Neuroblastoma

(e) Wilm’s tumour

A

(a) Mesoblastic nephroma

Mesoblastic nephroma is the most common solid intrarenal mass in the neonate. 90% present within the 1st year
(most by 3 months). It is benign, but there is a risk of sarcomatous degeneration·. Wilm’s tumour is commoner in
older infants; neuroblastoma is the commonest abdominal malignancy in neonates.

How well did you know this?
1
Not at all
2
3
4
5
Perfectly
44
Q

12 What is the commonest presenting symptom of Wilm’s tumour?

(a) Abdominal pain

(b) Haematuria

(c) Hypertension

(d) Palpable abdominal mass

(e) Weight loss

A

(d) Palpable abdominal mass

The commonest presenting features of Wilm’s tumour are: palpable mass (90%; 12 cm is the mean diameter at
diagnosis), hypertension (50%), pain (35%), anorexia (15%), fever (15%) and haematuria (5%).

How well did you know this?
1
Not at all
2
3
4
5
Perfectly
45
Q

19 Antenatal US shows bilateral renal pelvis dilation. A micturating cysto-urethrogram is performed. On the right
side there is reflux of contrast into the ureter, renal pelvis and calyces but no dilatation. On left side there is
mild/moderate dilatation of the ureter, renal pelvis and calyces with minimal blunting of the fornices.
What is the correct grading of the patient’s vesico-ureteric reflux?

(a) Right grade 0, left grade I

(b) Right grade I, left grade II

(c) Right grade II, left grade III

(d) Right grade Ill, left grade IV

(e) Right grade IV, left grade V

A

(b) Right grade I, left grade II

Grade I - reflux into ureter.
Grade II - reflux into the ureter/ pelvis/ calyces; no dilatation.
Grade III - moderate dilatation with minimal blunting of the fornices.
Grade IV - moderate ureteral tortuosity and obliteration of the fornices.
Grade V - gross dilatation with tortuous ureter.

How well did you know this?
1
Not at all
2
3
4
5
Perfectly
46
Q

25 An 18 month old boy presents with fever and intractable watery diarrhoea and is found to be hypertensive. On examination there is a palpable mass in the left side of the abdomen. US and subsequent CT examination reveal a mass superior to the left kidney which is noted to contain hypoechoic areas on US and low attenuation areas on CT. What is the most likely diagnosis?

(a) Adrenocortical carcinoma

(b) Adrenal haemorrhage

(c) Neuroblastoma

(d) Tuberculosis

(e) Wilm’s tumour

A

(c) Neuroblastoma

Although some of the imaging features may overlap between these conditions, the history is highly suggestive of
neuroblastoma. Intractable diarrhoea is secondary to elevated levels of VIP.’· Hypertension suggests the presence
of increased catecholamine (VMAand HVA) production.

How well did you know this?
1
Not at all
2
3
4
5
Perfectly
47
Q

(GU) 34 An otherwise normal baby is found to have an impalpable right testis at birth. At 6 months he undergoes an US to confirm the presence of an undescended testis. What is the most likely location for such a testis?

(a) Inguinal canal

(b) Pubic tubercle

(c) Immediately inferior to the right kidney

(d) Contralateral hemiscrotum

(e) Perineum

A

(a) Inguinal canal

The inguinal canal is the commonest site for an undescended testis, although it can lie anywhere along its
embryological passage from kidney to scrotum. Ectopic testes can be found in the femoral canal, perineum,
contralateral hemiscrotum or prepubic space. Localisation is important as, if present, there is a greatly increased
risk of malignancy

How well did you know this?
1
Not at all
2
3
4
5
Perfectly
48
Q

36 A neonate is found to have a left kidney measuring 5.4 cm, right kidney 4.3 cm. A subsequent IVU demonstrates a complete duplication of the left kidney. The patient returns for follow-up IVU late in childhood. Which of the following signs would be unexpected?

(a) Concave upper border of the left kidney

(b) Increased number of calyces in the left kidney compared to the right

(c) Obstruction of the ectopic ureter

(d) Upper Pole (ectopic) ureter inserting more inferior and media II

(e) Filling defect inferiorly within the bladder

A

(b) Increased number of calyces in the left kidney compared to the right

Asymmetrical renal size may be the only US finding with a duplex system. The ‘Weigert-Meyer’ rule states that
the lower pole ureter inserts in the normal position, the upper pole ureter inserts inferiomedially, tends to be
stenotic and obstructed and is often associated with a ureterocoele. The lower ureter is prone to VUR. The renal
pelvis will have a concave upper border due to inferior displacement by the · obstructed upper pole, there is lateral
displacement of the kidney and ureters and there are decreased calyces in comparison to the normal side,
producing the ‘drooping lily’ sign.

How well did you know this?
1
Not at all
2
3
4
5
Perfectly
49
Q

38 A 20 week antenatal US shows multiple cysts within the left kidney. These vary in size and are randomly
arranged. The parenchyma is difficult to demonstrate and the left renal pelvis cannot be identified. No other
anomalies are identified. What is the most likely diagnosis?

(a) Autosomal dominant polycystic kidney disease

(b) Autosomal recessive polycystic kidney disease

(c) Meckel-Gruber syndrome

(d) Multicystic dysplastic kidney

(e) Multilocular cystic nephroma

A

(d) Multicystic dysplastic kidney

MCKD is normally unilateral (bilateral in 5-20%) and is thought to be caused by pelvi-infundibular atresia. US
shows multiple cysts of variable size that are randomly arranged and are separated by little or no parenchyma; a
renal pelvis cannot be seen. It is the 2nd most common cause of a neonatal abdominal mass (after hydronephrosis)
and is the most common renal cystic disease in infants. Meckel-Gruber syndrome produces enlarged polycystic
(2-10 mm cysts) kidneys in utero, and typically causes oligohydramnios and pulmonary hypoplasia. MLCN is
characterised by large(> 10 cm) cystic spaces, which will compress or displace the parenchyma.

How well did you know this?
1
Not at all
2
3
4
5
Perfectly
50
Q

41 Which of the following is not a risk factor for neonatal renal vein thrombosis?

(a) Birth trauma

(b) Dehydration

(c) Diabetic mother

(d) Right adrenal haemorrhage

(e) Sepsis

A

(d) Right adrenal haemorrhage

Neonatal causes include dehydration (vomiting, diarrhoea), sepsis, birth trauma and prematurity. Glycosuria in
infants of diabetic mothers is another cause, ~md there is an association with increased maternal age. Left adrenal
haemorrhage is a cause of RVT; the lt adrenal vein drains to left RV, the right adrenal drains directly to IVC.

How well did you know this?
1
Not at all
2
3
4
5
Perfectly
51
Q

(GU) 49 A 32 year old male is found to have a complete duplex ureteric system on the left side at IVU. Which of the following statements is incorrect?

(a) The upper moiety ureter is more prone to obstruction

(b) Enuresis is less likely in female patients

(c) The lower moiety ureter is more prone to reflux

(d) The upper moiety ureter inserts inferior and medial to the lower moiety ureter

(e) The drorooping lily’ Sign may be seen on his IVU

A

(b) Enuresis is less likely in female patients

In men the upper moiety ureter always inserts proximal to the external urethral sphincter, thus enuresis is less
likely. By way of contrast, in females the upper moiety ureter commonly inserts distal to the sphincter, making
enuresis more likely. Drooping lily sign refers to IVU obstructed upper moiety ureter displacing lower ur inf.

How well did you know this?
1
Not at all
2
3
4
5
Perfectly
52
Q

(GU) 51 A 2 year old presents with pain and abdominal swelling. US shows an 8 cm hyperechoic poorly defined mass in the retroperitoneum, crossing the midline, with areas of acoustics shadowing within it. CT demonstrates
the presence of retroperitoneal adenopathy and encasement of the IVC. What is the likely diagnosis?

(a) Wilm’s tumour

(b) Rhabdomyosarcoma

(c) Renal cell carcinoma

(d) Neuroblastoma

(e) Polycystic kidney disease

A

(d) Neuroblastoma

Neuroblastoma is the commonest solid abdominal mass in infants. Itis characterised by difficulty in defining it
separately from the kidney, the presence of coarse stippled calcification and the fact it crosses the midline. It may
arise from any site within the sympathetic neural chain.

How well did you know this?
1
Not at all
2
3
4
5
Perfectly
53
Q

(GU) 62 A male infant is found to have multiple bladder diverticulae along with other congenital abnormalities.
Which of the following conditions is least likely?

(a) William’s syndrome

(b) Menke’s kinky hair syndrome

(c) Eagl Barrett syndrome

(d) Diamond-Blackfan syndrome

(e) Meckel-Gruber syndrome

A

(e) Meckel-Gruber syndrome

Meckel-Gruber syndrome is a cause of multiple renal cortical cysts but is invariably fatal at birth due to a
combination of lung hypoplasia and renal failure.

How well did you know this?
1
Not at all
2
3
4
5
Perfectly
54
Q

(GU) 64 A 50 year old man undergoes an IVU for suspected uretericcolic. No abnormality is found other than a
right ureter which swings medially at the level of L3/L4. Which of the following is the likeliest diagnosis?

(a) Para-aortic lymphadenopathy

(b) Neurofibroma

(c) Retrocaval ureter

(d) Pelvic lipomatosis

(e) Aortic aneurysm

A

(c) Retrocaval ureter

Retrocaval ureter is due to a rare embryological failure of IVC formation with persistence of the right posterior
cardinal vein and failure of the right supracardinal system to develop. It may be asymptomatic or cause varying
degrees of proximal hydroureter.

How well did you know this?
1
Not at all
2
3
4
5
Perfectly
55
Q
  1. A neonate has an abdominal ultrasound following the finding of a palpable abdominal mass. The right kidney
    is normal. The left kidney is replaced by multiple cysts of varying size and shape. There is no communication
    between the cysts, which are separated by septae. Which of the following is the most likely diagnosis?

a. Multilocular cystic renal tumour

b. Autosomal recessive polycystic kidney disease

c. Autosomal dominant polycystic kidney disease

d. Multicystic dysplastic kidney

e. Medullary sponge kidney

A
  1. d. Multicystic dysplastic kidney

These features are highly suggestive of multicystic dysplastic kidney. The condition is invariably fatal if bilateral,
but often asymptomatic if unilateral. It is the second commonest cause of a palpable abdominal mass in the
neonate, second only to hydronephrosis. The key features are that no normal renal tissue is present, the cysts do
not communicate with each other and they are separated by septae.

How well did you know this?
1
Not at all
2
3
4
5
Perfectly
56
Q
  1. A three year old boy has an ultrasound and an abdominal CT scan following the discovery of a palpable
    abdominal mass. No other symptoms are present. A large abdominal mass measuring 9 cm in maximum crosssectional dimensions is seen. Which one of the following features would favour a diagnosis of Wilms tumour
    rather than neuroblastoma?

a. Calcification

b. Bone metastases

c. Lung metastases

d. Encasement of the major vessels

e. Displacement of the kidney

A
  1. c. Lung metastases

Lung metastases are rare in neuroblastoma and are seen in only 10% of cases, whereas they are seen in 85% of
metastatic Wilms tumours. Wilms tumours tend to displace rather than encase the vessels, and intrinsic mass
effect, rather than displacement of the kidney, is seen. Calcification is seen in 90%of neuroblastomas but only
10% ofWilms tumours. Wilms tumours are the most common abdominal neoplasmin children between one and
eight years of age. It typically presents with an asymptomatic abdominal mass but pain, haematuria and fever
may be found.

How well did you know this?
1
Not at all
2
3
4
5
Perfectly
57
Q
  1. An eight month old girl is diagnosed with neuroblastoma following the finding of an abdominal mass. At CT
    the tumour is found to arise from the right suprarenal region and does not cross the midline. There are liver
    metastases, and bone marrow aspirates are positive for tumour. However, there is no evidence of skeletal
    metastases on plain films. What is the correct stage of the tumour?

a. Stage I

b. Stage II

c. Stage III

d. Stage IV

e. Stage IVs

A
  1. e. Stage IVs

The tumour should be staged as stage IVs. This stage refers specifically to patients less than one year of age with
no crossing of the midline and disease confined to the liver, skin and bone marrow without radiographically
evident skeletal metastases. It confers a good prognosis. Stage I is tumour confined to the organ of origin, stage
II includes regional spread not crossing the midline, stage III is extension across the midline and stage IV includes
metastatic disease.

58
Q
  1. A two year old boy presents with failure to thrive and a left-sided abdominal mass is found. An ultrasound
    scan reveals a left-sided hydronephrosis. The right kidney is normal. Which one of the following is most likely
    to be found as the underlying cause?

a. Posterior urethral valve

b. Pelvic–ureteric junction obstruction

c. Urethral diverticulum

d. Ureteric calculus

e. Ureterocoele

A
  1. b. Pelvic–ureteric junction obstruction

Hydronephrosis is a common cause of a palpable abdominal mass in children. The most common cause of
hydronephrosis is pelvic–ureteric junction obstruction, followed by posterior urethral valves and ectopic
ureterocoele. Pelvic–ureteric junction obstruction is usually due to a functional abnormality of the ureteric
musculature and is found more commonly on the left than the right.

59
Q
  1. A neonatal boy has a renal ultrasound performed for the investigation of urinary obstructive symptoms. The
    ultrasound shows a distended urinary bladder with bilateral hydronephrosis. Which one of the following is the
    most likely underlying pathology?

a. Posterior urethral valve

b. Neurogenic bladder

c. Horseshoe kidney

d. Ectopic ureterocoeles

e. Urethral diverticulum

A
  1. a. Posterior urethral valve

Posterior urethral valve is a congenital disorder characterised by a thick mucosal fold located in the posterior
urethra. It is the most common cause of bilateral urinary tract obstruction in boys. It is most commonly discovered
in the neonatal period, but very occasionally may present into adulthood. Diagnosis is usually made with
ultrasound and surgical treatment is indicated.

60
Q
  1. A two month old boy born prematurely has a CT abdomen following macroscopic haematuria and a palpable
    abdominal mass. A large intrarenal mass is seen, which replaces the majority of the renal parenchyma, involves
    the renal sinus and does not display any venous extension, collecting system involvement or calcification. Which
    one of the following diagnoses is most likely?

a. Metanephric adenoma

b. Mesoblastic nephroma

c. Wilms tumour

d. Nephroblastomatosis

e. Neuroblastoma

A
  1. b. Mesoblastic nephroma

Mesoblastic nephroma is the most likely diagnosis. This is the most common solid renal neoplasm in the neonate.
Although definitive diagnosis with CT or ultrasound is difficult, certain features make the diagnosis more likely; it displays infiltrative growth without involvement of the vascular structures or collecting system (differentiating
it from Wilms tumour) and only very rarely calcifies (differentiating it from neuroblastoma).

61
Q

(GU) 57. A 37 year old male undergoes an intravenous urogram and the right ureter is deviated medially in the
lumbar region. Which one of the following could explain this finding?

a. Psoas muscle hypertrophy

b. Para-aortic lymphadenopathy

c. Retrocaval ureter

d. Urinoma

e. Abdominal aortic aneurysm

A
  1. c. Retrocaval ureter

Medial deviation of the ureter is seen with retrocaval ureter on the right side and with retroperitoneal fibrosis.
The other conditions listed all cause medial deviation of the ureter in the lumbar region. Retrocaval ureter is a
rare entity which is caused by abnormal embryogenesis of the IVC. There may be symptoms of right ureteral
obstruction and recurrent urinary tract infections.

62
Q
  1. A three year old boy is seen in the outpatient department following recurrent urinary tract infections. Which
    one of the following imaging modalities would be most appropriate to detect the extent of renal scarring?

a. Tc-99m DTPA scintigraphy

b. Tc-99m DMSA scintigraphy

c. MAG-3 renogram

d. Micturating cystourethrography

e. I-131 OIH scintigraphy

A
  1. b. Tc-99m DMSA scintigraphy

DMSA scan is static renal scintigraphy. There is uptake in the proximal convoluted tubules with 50% uptake
within two hours and no significant excretion in this time. Excellent images of the cortex can therefore be gained,
making it a good investigation for detection of renal scarring. DMSA and MAG-3 provide more functional
information.

63
Q
  1. An 18 month old child with anaemia presents with E. coli gastroenteritis, heart failure and acute renal failure
    necessitating dialysis. Blood screen also confirms thrombocytopaenia. Which of the following is the most likely
    diagnosis?

a. Pelvic–ureteric junction obstruction

b. Medullary sponge kidney

c. Autosomal recessive polycystic disease

d. Haemolytic-uraemic syndrome

e. Medullary cystic disease

A
  1. d. Haemolytic-uraemic syndrome

Haemolytic-uraemic syndrome is the commonest cause of acute renal failure in children needing dialysis.

64
Q

QUESTION 7 A 2-year-old boy presents with repeated urinary tract infections. Which one of the following
investigations would be the most appropriate investigation?

A Intravenous urogram at 6 weeks

B Micturating cystogram at 6 weeks

C Tc-99m DMSA renal scintigraphy at 4 months

D Tc-99m DTPA indirect cystography at 4 months

E An ultrasound at 4 months

A

ANSWER: C

A micturating cystogram is indicated if the infant is under the age of 6 months, but is not a requirement if the infant is > 6 months unless there is an abnormal US or DMSA.

65
Q

(GU) QUESTION 8
A full-term neonate has a palpable left-sided abdominal mass. On ultrasound, the right kidney appears normal
whilst the left kidney is grossly enlarged, containing multiple anechoic cysts of varying sizes which do not
communicate with each other. A subsequent MAG3 scintigram confirms normal function within the right kidney
and no evidence of isotope uptake on the left. What is the most likely underlying diagnosis?

A Autosomal dominant polycystic kidney disease (ADPKD)

B Autosomal recessive polycystic kidney disease (ARPKD)

C Hydronephrosis

D Infantile form of medullary sponge kidney (IMSK)

E Multicystic dysplastic kidney disease (MCKD)

A

E Multicystic dysplastic kidney disease (MCKD)

66
Q

QUESTION 13
On an antenatal ultrasound of a male fetus, bilateral hydronephrosis was identified. A subsequent ultrasound
demonstrates worsening hydronephrosis with a full bladder and reduced volume of amniotic fluid. As a result
labour is induced early. Which one of the following postnatal investigations will provide a definitive diagnosis?

A A repeat postnatal ultrasound

B Cystography

C DMSA renal scintigraphy

D Micturating cystourethrography

E Tc-99m MAG scintigram

A

D Micturating cystourethrography

The history suggests posterior urethral valves. The most appropriate first line imaging is a micturating
cystourethrogram with oblique views.

67
Q

QUESTION 14 A 6-month-old child with a palpable abdominal mass is initially investigated with an ultrasound,
revealing a mixed echogenic mass in the left kidney. A subsequent CT demonstrates a large mass within the left
kidney which has a moderately enhancing component. Which one of the following would be likely diagnosis?

A Angiomyolipoma

B Lymphoma

C Nephroblastomatosis

D Neuroblastoma

E Wilms’ tumour

A

E Wilms’ tumour

Wilms’ tumours are associated with nephroblastomatosis; however, nephroblastomatosis is nonenhancing

68
Q

@#e (GU) QUESTION 15
A 29-year-old man has an IVU performed following an episode of haematuria. This demonstrates complete
right-sided ureteric duplication. Which one of the following statements is true?

A If present, an ectopic ureterocoele is usually related to the lower moiety

B The lower moiety ureter usually obstructs at the vesicoureteric junction.

C The upper moiety calyces are prone to vesicoureteric reflux.

D The upper moiety ureter is prone to ureteric obstruction.

E The upper moiety ureter usually inserts into the bladder superior to the lower moiety ureter

A

D The upper moiety ureter is prone to ureteric obstruction.

69
Q

QUESTION 31
A neonate presents with an abdominal mass. An abdominal radiograph reveals normal bowel gas pattern with no
calcification. Ultrasound demonstrates a cystic mass in the left flank. Which one of the following conditions is
most likely in this neonate?

A Hydronephrosis

B Infantile polycystic kidney

C Mesonephric nephroma

D Multicystic dysplastic kidney

E Wilms1 tumour

A

A Hydronephrosis

Over half of abdominal masses originate from the genitourinary tract. Hydronephrosis makes up 25% of these. A multicystic dysplastic kidney makes up about 15%; the remaining diagnoses rarely present as cystic masses.

70
Q

QUESTION 37
On a routine 20-week antenatal scan, the right fetal kidney contains multiple large cysts varying in size and
scattered in a random distribution. No discernible renal tissue is seen. The left kidney appears normal. What is
the most likely diagnosis?

A Autosomal recessive polycystic kidney disease

B Medullary sponge kidney

C Multicystic dysplastic kidney

D Multiple simple cysts

E Tuberous sclerosis

A

C Multicystic dysplastic kidney

The fact that the condition is unilateral and detected during the antenatal scan makes C the most likely diagnosis.
The remaining diagnoses are most often bilateral or diagnosed later in life.

71
Q

QUESTION 39
A child attends the Medical Physics Department for a Tc-99m DTPA renal isotope study. Which one of the
following statements is a justified indication for the scan?

A Assessment of renal scarring

B Establishing divided renal function

C Follow-up of pyelonephritis

D Investigating recurrent urinary tract infections

E Suspected duplex kidney

A

B Establishing divided renal function

Indications for a DTPA scan also include follow-up of renal transplant, renography with captopril stimulation and
postoperative evaluation of a collecting system.

72
Q

QUESTION 46
On a 20-week antenatal ultrasound, unilateral fetal hydronephrosis is detected. Which one of the following
findings would confirm the diagnosis of renal pelvic dilatation (RPD)?

A During the second trimester, the AP renal pelvis measures more than 3 mm.

B During the third trimester, the AP renal pelvis measures greater than 5 mm.

C Megaureters are present.

D The AP renal pelvis measures greater than 25% of the longitudinal length of the kidney.

E The AP renal pelvis measures greater than 50% of the longitudinal length of the kidney.

A

E The AP renal pelvis measures greater than 50% of the longitudinal length of the kidney.

RPD is diagnosed when there is calyceal dilatation with a renal pelvis that measures greater than 5 mm in the
second trimester or 10 mm in the third trimester. No ureteric dilatation should be seen.

73
Q

QUESTION 49
A 4-year-old child presents with groin pain and haematuria. An ultrasound is performed and suggests crossed
fused ectopic kidney. Which one of the following statements is true regarding the anatomy of renal anomalies?

A In crossed fused ectopia, both of the kidneys are abnormally placed.

B In crossed fused ectopia, both ureters enter the bladder in a normal Position

C In crossed fused ectopia, the crossed kidney is fused superiorly to the

D The upper poles are fused in a horseshoe kidney.

E The ureters pass posterior to the lower poles in a horseshoe kidney.

A

B In crossed fused ectopia, both ureters enter the bladder in a normal Position

Horseshoe kidney is the most common fusion abnormality and arises when both lower poles have fused together.
Crossed fused ectopia occurs when one kidney is displaced across the midline and fused inferiorly to the other
kidney, but both ureters enter the bladder in a normal position

74
Q

QUESTION 55
A 1-year-old girl is being investigated for repeated urinary tract infections. She undergoes an ultrasound which shows a left duplex kidney with a dilated upper collecting system and a large left ureterocoele. Which one of the following statements is true?

A Ectopic drainage of the ureter is rarely associated with scarring of the kidney.

B The ureter draining the lower moiety usually enters the bladder as a ureterocoele.

C The ureter draining the upper moiety usually enters the bladder as a ureterocoele.

D The ureter draining the upper moiety is usually affected by reflux.

E When one moiety drains outside the bladder, it is usually the lower pole.

A

C The ureter draining the upper moiety usually enters the bladder as a ureterocoele.

The upper ureter usually enters as an ureterocele, but the lower pole ureter is the one more frequently affected by
reflux. Reflux is rarely seen to the upper moiety.

75
Q

UESTION 64
A 3-year-old child presents with weight loss and an abdominal mass. On the initial ultrasound, a large mass is
seen in the region of the right kidney/ adrenal. A CT abdomen is arranged. Which one of the following statements
is correct when differentiating between a Wilms’ tumour and neuroblastoma?

A Neuroblastoma enhances more than the adjacent renal parenchyma.

B Neuroblastoma usually displaces rather than encases vessels.

C Pulmonary metastases are more suggestive of Wilms’ tumour.

D Skeletal metastases are more suggestive of Wilms’ tumour.

E Wilms’ tumour is more likely to contain calcification.

A

C Pulmonary metastases are more suggestive of Wilms’ tumour.

Calcification on CT is seen in up to 85% of children with neuroblastoma and bone metastases are also more
common with this tumour. It is usual for both tumours to enhance less than the surrounding renal parenchyma
and neuroblastomas tend to encase vessels, rather than displace them

76
Q
  1. A 2-day-old term neonate with an antenatal history of enlarged kidneys undergoes ultrasound of the
    renal tracts, which reveals bilateral enlarged and diffusely echogenic kidneys, with loss of corticomedullary differentiation. Further assessment with a high-resolution linear probe reveals multiple small
    radially oriented cysts. Which of the following statements regarding this condition is false?

A. It is associated with congenital hepatic fibrosis.

B. The severities of renal and hepatic involvement are inversely proportional to each other.

C. Severe renal compromise is the immediate cause of death in the perinatal group.

D. Potter facies may be found in severely affected neonates.

E. It is associated with clubfoot deformity

A
  1. C. Severe renal compromise is the immediate cause of death in the perinatal group.

The incidence of autosomal recessive polycystic kidney disease (ARPKD) is approximately 1:20,000 births. It is a disease of tubular ectasia and fibrosis that results in bilateral enlarged kidneys, with loss of cortico-medullary differentiation and multiple small radially arranged cysts. ARPKD is associated with congenital hepatic fibrosis, the severity of which is inversely proportional to the renal abnormality.
Four distinct groups of ARPKD are recognized based on age at presentation: perinatal, neonatal, infantile, and juvenile. The most severe renal involvement is seen in the perinatal group. In this group, severe renal impairment results in reduced urine output, oligohydramnios, and pulmonary hypoplasia. Severe respiratory compromise is the immediate cause of death in these patients. Oligohydramnios is also associated with Potter facies (low set and
flattened ears, short and snubbed nose, deep eye creases, and micrognathia) and clubfoot deformity.

77
Q
  1. A full-term infant with unilateral cryptorchidism is referred for ultrasound assessment. Which is the
    most common location of a cryptorchid testis?

A. Inguinal canal.

B. Superficial inguinal ring.

C. Deep inguinal ring.

D. Femoral triangle.

E. Abdomen.

A
  1. A. Inguinal canal.

The testes form in the retroperitoneum and descend through the inguinal canal into the scrotum. Failure to descend
is known as cryptorchidism. A cryptorchid testis may be located at any point along the descent route. The most
common location is the inguinal canal (70% of cases) followed by the prescrotal region, just beyond the
superficial inguinal ring. On ultrasound, it is usually smaller and hypo or isoechoic compared to the normally
located testis. Cryptorchidism is associated with an increased risk of malignancy even after correction.

78
Q
  1. A 32-year-old male has been referred from urology for assessment. The patient was involved in an RTA
    2 years ago, during which he sustained a urethral injury. The patient had failed to attend urology
    outpatients for follow-up in the interim, but has re-attended with recurrent UTIs. An attempted cystoscopy
    identified a tight proximal urethral stricture. The urologists have requested an ultrasound and voiding
    cystourethrogram (VCUG) to investigate the degree of bladder outlet obstruction. The ultrasound
    demonstrates a mild degree of renal pelvic dilatation bilaterally. The bladder has a residual volume of 500
    ml on post-void imaging. The VCUG reveals reflux into the renal pelvis bilaterally with mild ureteric and
    pelvic dilatation, but no calyceal dilatation and preserved forniceal angles. What grade of reflux does this
    patient have?

A. Grade 1.

B. Grade 2.

C. Grade 3.

D. Grade 4.

E. Grade 5

A
  1. C. Grade 3.

Whilst most commonly associated with congenital reflux in children, acquired reflux is also seen, most commonly
due to bladder outlet obstruction or recurrent cystitis. Grade 1 reflux demonstrates reflux into the ureter, grade 2
has reflux into the pelvicalyceal system, grade 3 as described, and grade 4 is as grade 3 but with increased
pelviureteric dilatation and calyceal blunting, and with persisting papillary impressions. Grade 5 demonstrates a
markedly dilated pelvis and tortuous ureter with obliteration of the forniceal angles and papillary impressions.

79
Q
  1. A 7-year-old girl presents with a history of continuous dribbling incontinence. On imaging she is found
    to have bilateral duplex kidney, complete ureteral duplication, and ectopic ureter. Which of the following
    statements regarding ectopic ureters and ureteral duplication is true?

A. Ectopic insertion is more commonly associated with solitary ureter than complete ureteral duplication.

B. Urinary incontinence with ectopic ureter is more common in boys.

C. In complete ureteral duplication, the ureter of the upper pole moiety inserts into the bladder superior to the ureter of lower pole moiety.

D. The upper moiety ureter is associated with ureterocele whereas the lower moiety ureter is associated with vesicoureteric reflux.

E. The obstructed lower moiety may not be visualized on IVU.

A
  1. D. The upper moiety ureter is associated with ureterocele whereas the lower moiety ureter is associated with vesicoureteric reflux.

Ectopic ureter results when the ureteral bud fails to separate from the Wolffian duct and as a consequence is
carried more caudally than normal. In females, the ectopic ureter can insert distal to the external sphincter, resulting in incontinence. Approximately 70% of ectopic insertion is associated with complete ureteral duplication. According to the Weigert–Meyer rule of complete ureteral duplication, the ureteric orifice of the upper moiety inserts into the bladder medial and inferior to the lower moiety. The ureter draining the Upper moiety is associated with Ureterocele and Obstruction (note the vowels), whereas the lower pole moiety is associated with vesico-ureteric reflux. The obstructed upper moiety may not be visualized on IVU. The absence
of upper pole calyx and displaced (‘drooping lily’) lower moiety calyces help in making the diagnosis.

80
Q

(GU) 12. A 2-day-old male infant with cryptorchidism and an antenatal diagnosis of dilated bladder and
ureters is referred for a micturating cystourethrogram (MCUG). MCUG reveals a dilated bladder,
tortuous and dilated ureters, dilated posterior urethra, and renal cortical thinning. Note is also made of
bulging flanks. What is the diagnosis?

A. Posterior urethral valve.

B. Congenital megacystis and megaureter.

C. Bilateral vesicoureteric reflex.

D. Eagle Barrett syndrome.

E. Meatal stenosis.

A
  1. D. Eagle Barrett syndrome.

Eagle Barrett syndrome or prune belly syndrome is classically defined as a triad of partial or complete absence
of abdominal musculature (resulting in bulging flanks), cryptorchidism, and urinary abnormalities as described
above. In addition a number of respiratory, GI, musculoskeletal, and cardiovascular anomalies are associated with
this syndrome. It occurs almost exclusively in males. The cause and embryogenesis remain controversial. Bladder
outlet obs, mesodermal arrest, and dysgenesis of the yolk sac have been proposed as possible causes.

81
Q
  1. A child is diagnosed with neuroblastoma. He is referred for staging and you are asked to advise on the
    standard radiological investigation of bony metastases. What do you advise?

A. Whole body MRI.

B. Whole body 18-FDG PET-CT.

C. 123I-metaiodobenzylguanidine (MIBG) scan.

D. 99mTc methylene-diphosphonate (MDP) isotope bone scan.

E. MIBG and isotope bone scan.

A
  1. C. 123I-metaiodobenzylguanidine (MIBG) scan.
    Owing to the high specificity and sensitivity in neuroblastoma, MIBG imaging has superseded the use of 99mTc bone scans for the detection of skeletal metastases in the majority of children with neuroblastoma, which take up
    the tracer in >90% of cases, and has been recommended by the last international consensus conference as a standard element of staging and response evaluation. False-negative scans may be observed in approximately
    10% of neuroblastomas that do not concentrate MIBG. In addition, very small amounts of bone marrow tumour
    will often not be detected and therefore the MIBG scan must be supplemented with bilateral bone marrow biopsy.
    For those patients whose tumours are negative for MIBG uptake at diagnosis, the 99mTc MDP bone scan is the
    standard test recommended to evaluate skeletal metastases. However, the low specificity of this test and the
    difficulty in interpreting uptake in young children with actively growing bones make investigation of alternative
    methods preferable. Whole-body MRI is also a sensitive test for neuroblastoma tumours, including bone and bone
    marrow metastases, although the specificity is much lower than for MIBG. MRI and CT are appropriate in the
    staging of neuroblastoma, but for the assessment of local invasion rather than bony involvement in particular.
    There currently is no consensus about the optimal imaging modality for assessing local disease. Both MRI and
    CT are routinely used, depending on local availability and the radiologist’s preference. 18-FDG PET-CT imaging
    is not as sensitive as MIBG imaging for bony metastases, although it may be more sensitive for small soft tissue
    tumours & nodal metastases. The use of PET-CT is not at present well defined and delivers a high radiation dose.
82
Q
  1. A 6-week-old infant presents with a history of failure to thrive. Plain abdominal radiograph
    demonstrates punctate calcification in the region of the adrenal glands. Which of the following findings on
    CT of abdomen is most specific for Wolman disease?

A. Hepatosplenomegaly.

B. Enlarged retroperitoneal lymph nodes.

C. Diffuse fatty infiltration of the liver.

D. Enlarged calcified adrenals that maintain their normal triangular configuration.

E. Thickening of small bowel.

A
  1. D. Enlarged, calcified adrenals that maintain their normal triangular configuration.

Wolman disease is a rare, autosomal recessive, primary familial xanthomatosis with involvement and calcification
of the adrenal glands. Deficiency of acid esterase or acid lipase results in lipid deposition in multiple organs,
including the liver, spleen, lymph nodes, adrenal cortex, and small bowel. It has a poor prognosis. Most patients
die within 6 months of life due to malabsorption. All the above-mentioned features are noted in Wolman disease
but the most specific finding is enlarged, calcified adrenal glands that maintain their normal configuration.

83
Q
  1. An antenatal ultrasound of foetus at 20 weeks gestation reveals an occipital encephalocele. Foetal MRI
    demonstrates bilateral enlarged kidneys with cystic dysplasia and polydactyly. What is the diagnosis?

A. Autosomal recessive polycystic kidney disease.

B. Bardet-Biedl syndrome.

C. Meckel Gruber syndrome.

D. Tuberous sclerosis.

E. Zellweger syndrome.

A
  1. C. Meckel Gruber syndrome.

All of the mentioned conditions are associated with multiple renal cysts, but the triad of bilateral enlarged cystic
kidneys, occipital encephalocele, and polydactyly is diagnostic of Meckel Gruber syndrome. Bardet-Biedl
syndrome is associated with enlarged cystic dysplasia of the kidneys with polydactyly.

84
Q

(GU) 52. A 2-day-old male neonate with a right-sided abdominal mass is referred for ultrasound of
abdomen. Ultrasound demonstrates an enlarged right kidney containing multiple non-communicating
cysts of varying size with little normal parenchyma. What is the most common associated abnormality of
the contra-lateral kidney?

A. Ectopic ureter.

B. Pelvi-ureteric junction obstruction.

C. Vesico-ureteric reflux.

D. Renal hypoplasia.

E. Renal aplasia.

A
  1. C. Vesico-ureteric reflux.

The abnormality described on ultrasound is multicystic dysplastic kidney (MCDK). It is the most common form
of cystic disease in infants and the second most common cause of an abdominal mass in a neonate (after
hydronephrosis). Obstruction/atresia of the ureter during the developmental stage is thought to be the etiology.
Bilateral MCDK is uncommon, but associated anomalies of the contra-lateral kidney are seen in up to 50% of
cases. Vesico-ureteric reflux (30–40%) is the most common associated anomaly, followed by pelvi-ureteric
junction obstruction (10–20%)

85
Q
  1. A 5-year-old girl presents with a pelvic mass with extension into the gluteal region through the sciatic
    foramen. Which of the following statements regarding sciatic foraminal anatomy is correct?

A. Piriformis muscle forms the boundary between the greater and lesser sciatic foramen.

B. The sacrotuberous ligament divides the greater sciatic foramen into superior and inferior compartments.

C. The pudendal nerve and internal pudendal vessels pass through the lesser sciatic foramen.

D. The sciatic nerve passes through the superior part of greater sciatic foramen.

E. The superior gluteal vessels and nerve pass through the inferior part of the greater sciatic foramen.

A
  1. C. The pudendal nerve and internal pudendal vessels pass through the lesser sciatic foramen.

There are several potential pathways of extrapelvic spread of pelvic disease. Lateral extension into the gluteal
region occurs through the sciatic foramina. It is important to identify extrapelvic spread, as combined transabdominal and trans-gluteal surgery may be required. Knowledge of gluteal anatomy is also essential for transgluteal biopsy and drainages. The greater and lesser sciatic notches are noted in the posterior border of the
innominate bone separated by the ischial spine. The greater sciatic foramen (GSF) is bounded antero-superiorly
by the greater sciatic notch, postero-superiorly by the sacrum, and inferiorly by the sacrospinous ligament. The
piriformis muscle arises from the ventral aspect of the sacrum and passes through the GSF to insert into the greater
trochanter. It divides the GSF into two compartments. The suprapiriformis foramen transmits the superior gluteal
vessels and nerves. The infrapiriformis foramen transmits theinferior gluteal vessels and nerves, pudendal and
sciatic nerves. The lesser sciatic foramen (LSF) is bounded anteriorly by the lesser sciatic notch, superiorly by
the sacrospinous ligament, and postero-inferiorly by the sacrotuberous ligament. The obturator internus tendon,
the pudendal nerve, and the internal pudendal vessels pass through the LSF.

86
Q

(GU) 57. A 12-month-old infant with a history of aniridia and nephroblastomatosis undergoes a follow-up
CT of abdomen that demonstrates bilateral enlarged kidneys with a thick rind of homogenous, nonenhancing, hypodense tissue bilaterally. A focal heterogenous enhancing mass with cystic change is noted
on one side. What is the diagnosis?

A. Neuroblastoma.

B. Wilms tumour.

C. Lymphoma.

D. Renal cell carcinoma.

E. Cystic nephroma.

A
  1. B. Wilms tumour.

Sporadic aniridia is associated with nephroblastomatosis (multiple nephrogenic rests) and an increased risk of
Wilms tumour. At CT, the nephrogenic rests appear as homogenous, hypodense nodules/peripheral rind, with
little or no enhancement compared to the compressed normal cortical tissue. Any new, enlarging, or heterogenous
mass in the setting of nephroblastomatosis indicates the development of a Wilms tumour.

87
Q
  1. A 6-year-old boy undergoes an ultrasound examination of the renal tracts that shows dilated, polygonal,
    multifaceted calcyces in the right kidney. The infundibula, renal pelvis, and ureter are normal in calibre.
    IVU confirms the ultrasound findings and there is normal contrast excretion. What is the diagnosis?

A. Congenital megacalcyces.

B. Obstructive hydronephrosis.

C. Multicystic dysplastic kidney.

D. Congenital megacystis-microcolon syndrome.

E. Vesico-ureteric reflux.

A
  1. A. Congenital megacalyces

This is a rare developmental abnormality of the renal medulla. Imaging demonstrates dilated, multifaceted, or
polygonal calyces as opposed to spherical calyces of obstructive hydronephrosis. In addition, the infundibula,
renal pelvis, and ureter are normal in calibre in congenital hydronephrosis. The affected kidneys may demonstrate
cortical thinning but the renal function is preserved.

88
Q
  1. A 15-year-old male undergoes a pelvic MRI on which an incidental note is made of absent bilateral
    seminal vesicles. Which of the following is commonly associated with bilateral seminal vesicle agenesis?

A. Renal agenesis.

B. CF.

C. Calcified vas deferens.

D. Ectopic ureter.

E. Rotation anomaly of one kidney

A
  1. B. CF.

Bilateral seminal vesicle agenesis is frequently associated with bilateral agenesis of the vas deferens and
mutations in the CF transmembrane conductance regulator gene (64-73%). It is thought to be related to luminal
obstruction by thick secretions. Bilateral agenesis of the vas deferens is seen in 99% of CF in males. The affected
patients usually have normal kidneys. Unilateral seminal vesicle agenesis and seminal vesicle cyst are commonly
associated with ipsilateral renal agenesis. Bilateral calcified vas deferens is commonly seen in diabetics.

89
Q

5 A renal ultrasound on a five-year-old shows bilateral hyperechoic smoothly enlarged kidneys with a loss of
corticomedullary differentiation and a few macroscopic (<1 cm) renal cysts. Which of the following is the most
important additional investigation?

a Liver ultrasound

b Chest radiograph

C Micturating cystourethrogram

d Abdominal PET-CT

e Post-void bladder ultrasound

A

5 Answer A: Liver ultrasound

The description is that of autosomal-recessive polycystic kidney disease. This is associated with congenital
hepatic fibrosis. The less severe the renal involvement, the more severe the hepatic involvement.

90
Q

12 A four-year-old child presents with recurrent urinary tract infections. They have had three culture confirmed
urinary tract infections with no evidence of pyelonephritis in any episode. What is the most appropriate
investigation?

a Ultrasound scan during an episode of infection

b Ultrasound scan within six months

c DMSA in four to six months

d DMSA during acute infection

e Micturating cysto-urethrogram at a time when symptom free

A

12 Answer C: DMSA in four to six months

NICE guidelines on management of UTI recommend ultrasound within six weeks and a DMSA scan at four to
six months for investigation of recurrent UTI in children over three years and makes very similar
recommendations for children six months to three years with recurrent UTIs.

91
Q

13 A two-year-old child presents with increasing abdominal girth. An ultrasound shows a 13-cm mass in the
upper abdomen, arising on the right side and crossing the midline. A CT shows speckled calcification within it
and a preliminary diagnosis of neuroblastoma is made. What is most appropriate radionuclide study in this case?

a 1123 MIBG study with planar and SPECT images

b Inl l l octreotide study planar and SPECT images

c MDP Tc-99m bone scan whole body views

d Tc-99m pertechnetate study with planar images of the whole abdomen at 24 and 48 hours

e Whole body I123

A

13 Answer A: 1123 MIBG study with planar and SPECT images

Although neuroblastoma can be detected on octreotide studies MIBG is the first line of investigation. Whole body
images with I12 3 are used in the treatment & assessment of thyroid malignancy and have no place in this scenario.

92
Q

14 An infant is diagnosed with Beckwith-Wiedemann syndrome and is therefore at increased risk of developing
Wilms’ tumour. How should they be followed up?

a Abdominal ultrasound every three months

b Annual abdominal ultrasound

c Abdominal ultrasound at six months with further imaging only if the child becomes symptomatic

d Annual abdominal CT

e Abdominal CT every three months

A

14 Answer A: Abdominal ultrasound every three months

Ten to twenty per cent of patients with Beckwith-Wiedemann syndrome develop Wilms’ tumour. There is also an
increased risk of developing Wilms’ tumour in patients with sporadic aniridia, hemihypertrophy and genitourinary disorders such as Drash syndrome (male pseudohermaphroditism and glomerulonephritis).

93
Q

24 A three-month-old boy undergoes a renal tract ultrasound. This demonstrates a thick-walled bladder with
trabeculations and bilateral hydroureteronephrosis. Which of the following is the most likely diagnosis?

a Pelvi-ureteric junction obstruction

b Megacystis-microcolon-intestinal hypoperistalsis syndrome

C Posterior urethral valve

d Vesico-ureteric junction obstruction

e Prune belly syndrome

A

24 Answer C: Posterior urethral valve

These are congenital thick folds of mucosa within the posterior urethra. They are the most common cause of
urinary tract obstruction and end-stage renal disease in boys.

94
Q

28 An eight-year-old girl has never been continent of urine and complains of intermittent dribbling of urine,
particularly when she is upright. Which of the following is the most likely diagnosis?

a Complete ureteric duplication with infrasphincteric insertion of the upper moiety ureter

b Complete ureteric duplication with suprasphincteric insertion of the upper moiety ureter

C Partial ureteric duplication

d Pseudoureterocoele

e Congenital urethral diverticulum

A

28 Answer A: Complete ureteric duplication with infrasphincteric insertion of the upper moiety ureter

In complete ureteric duplication, there is an increased risk of obstruction of the upper moiety ureter and of vesicoureteric reflux of the lower moiety ureter. The upper moiety ureter has a suprasphincteric insertion in boys,
resulting in no enuresis but epididymitis and urge incontinence. In girls, the upper moiety ureter has an
infrasphincteric insertion and causes both wetting in upright position & intermittent or constant dribbling of urine.

95
Q

47 A two-year-old presents with an abdominal mass. He is found to be hypertensive and to have high levels of
vanillylmandelic acid in his urine. A CT scan shows a heterogeneous mass containing calcification arising from
the left adrenal gland and extending across the midline. There is no evidence of metastatic disease on the CT.
Appearances are suggestive of an adrenal neuroblastoma. What is the most appropriate staging?

a Stage I

b Stage II

C Stage III

d Stage IV

e Stage IVs

A

47 Answer C: Stage III

Neuroblastoma staging:
Stage I: Limited to organ of origin
Stage II: Regional spread, not crossing the midline
Stage III: Extending across the midline
Stage IV Metastatic spread to liver, bone, brain, lung, distant lymph nodes
Stage IVs: Stages I and II, with disease confined to the skin, bone marrow and liver with no radiographic evidence
of skeletal metastases

96
Q

(GU) 53 A 33-year-old female investigated for recurrent urinary infections is demonstrated to have a complete left duplex ureter on intravenous urography. Which of the following describes the classical arrangement of the duplex ureteric insertions and associated features?

a The ureter from the superior pelvis inserts above that from the inferior pelvis, and the upper renal moiety has calyceal dilatation from distal ureteric obstruction

b The ureter from the superior pelvis inserts below that from the inferior pelvis, and the upper renal moiety has calyceal dilatation from distal ureteric obstruction

c The ureter from the superior pelvis inserts below that from the inferior pelvis, and is more prone to vesicoureteric reflux

d The ureter from the inferior pelvis is associated with a ureterocoele

e The ureter from the inferior pelvis is associated with ectopic insertion into the urethra

A

53 Answer B: The ureter from the superior pelvis inserts below that from the inferior pelvis, and the upper renal moiety has calyceal dilatation from distal ureteric obstruction

The upper moiety ureter classically inserts ectopically low, below that of the lower moiety, and it is often
associated with a stenotic insertion site, which results in dilatation of the intramural ureter, forming an ectopic
ureterocoele and subsequent distal ureteric obstruction. The lower moiety ureter has a more horizontal path and
is more prone to reflex. RULE: Upper moiety inserts low and obstructs. Lower moiety refluxes.

97
Q

(GU) 56 A 15-year-old boy being investigated for recurrent urinary infection undergoes Tc-99m DTPA
scintigraphy which demonstrates the presence of a horseshoe kidney. What is likely to limit the isthmus?

a Umbilical ligament

b Urachus

c Coeliac axis

d Superior mesenteric artery

e Inferior mesenteric artery

A

56 Answer E: Inferior mesenteric artery

The inferior mesenteric artery limits the ascent of the isthmus, and the pelves lie anteriorly. Renal calculi are more common. Risk of TCC (transitional cell carcinoma) is three to four times the general population. Incidence of
horseshoe kidney is 1-4:1000.

98
Q

66 A left multicystic dysplastic kidney is diagnosed on an antenatal ultrasound. Which of the following are the
most likely findings in the left kidney on a post-natal ultrasound at three weeks?

a A 2-cm solitary cortical cyst within the upper pole

b Multiple cysts of varying sizes, an absent renal pelvis and dysplastic renal parenchyma

c A small kidney with poor corticomedullary differentiation and medullary cysts

d Enlarged hyperechoic kidney with poor corticomedullary differentiation and several cortical cysts

e Atrophic kidney measuring 1.5 cm in length

A

66 Answer B: Multiple cysts of varying sizes, an absent renal pelvis and dysplastic renal parenchyma

Calcifications may be seen in the cyst walls or in the septations between the cysts. The contralateral kidney often
shows compensatory hypertrophy.

99
Q

(GU) 67 A one-year-old male under investigation for an abdominal mass is demonstrated on ultrasound to have
multiple left renal cysts. There is no functioning ipsilateral renal tissue on MAG3 scintigraphy. What is the most
likely diagnosis?

a Acquired renal cystic disease

b Multicystic dysplastic kidney

C Autosomal recessive polycystic kidney disease

d Pelvi-ureteric junction obstruction

e Multiple simple cysts

A

67 Answer B: Multicystic dysplastic kidney

Multicystic dysplastic kidney is a congenital, non-hereditary renal dysplasia in which there is no functioning renal tissue, instead multiple thin-walled cysts and connective tissue. If bilateral it is fatal at birth.

100
Q

6 A 13-year-old boy has short stature with a webbed neck. He has a delayed bone age and an abdominal ultrasound
shows bilateral renal duplication. What is the most likely diagnosis?

a Di George syndrome

b Turner’s syndrome

C Cri du Chat syndrome

d Noonan’s syndrome

e Edward’s syndrome

A

6 Answer D: Noonan’s syndrome

Noonan’s $ has a phenotype similar to Turner’s syndrome but with a normal karyotype. It occurs in both sexes.

101
Q

(GU) 7 A two-year-old boy who presented with an abdominal mass and underwent ultrasound and CT
examination has been diagnosed with neuroblastoma. Which of the following radiological findings would indicate
that the lesion could not be fully removed surgically?

a Necrosis within the mass

b Displacement of the ipsilateral kidney

C Enlarged ipsilateral lymph nodes containing tumour

d Calcification within the mass

e Involvement of IVC

A

7 Answer E: Involvement of the IVC

Regional lymph nodes are removed at surgery. Caval involvement is an indicator of unresectability

102
Q

(GU) 6 A two-year-old boy presents to the paediatric team with an abdominal mass. Ultrasound examination demonstrated a large hyperechoic heterogeneous mass on the right extending across the midline and with areas
with acoustic shadowing. The right kidney could not be clearly visualised. A CT examination was then performed. Which of the following are the most likely radiological findings?

a Centrally located heterogeneous mass arising from the right kidney with a prominent peripheral crescent-shaped subcapsular fluid collection

b Solid intrarenal mass of right kidney with uniform enhancement of less than normal renal parenchyma with areas of low attenuation within it

c Heterogeneous solid mass with low-density areas & calcifications displacing the right kidney and encasing IVC

d Well-defined heterogeneous partially cystic mass with a claw of right renal tissue partially extending around it

e Well-defined round mass separate to the right kidney with periadrenal fat stranding and a low attenuation centre

A

6 Answer C: Heterogeneous solid mass with low density areas and calcifications displacing the right kidney and encasing the IVC

These findings are typical for neuroblastoma, which contains calcifications on CT in 85%, can extend across the midline and encases rather than invades vascular structures. Option (a) describes a rhabdoid tumour of the kidney,
option (b) mesoblastic nephroma (although 90% of these occur before 1 year), option (d) Wilms’ tumour and
option (e) adrenal haemorrhage.

103
Q

10 A male neonate is found to have a wrinkled hypotonic abdominal wall and no palpable testes within the scrotum. What is the most important radiological investigation to perform in the first instance?

a Abdominal radiograph

b Renal tract ultrasound

c Scrotal ultrasound

d Chest radiograph

e Micturating cystourethrogram

A

10 Answer B: Renal tract ultrasound

Prune belly syndrome is a congenital non-hereditary disorder that almost exclusively affects males. It is a triad of
abdominal wall muscle insufficiency, nonobstructed distended ureters (+/hydronephrosis and a degree of renal
dysplasia) and bilateral undistended testes.

104
Q

22 A renal ultrasound of a one month old shows a grossly abnormal left kidney with the normal renal architecture replaced by cysts of varying sizes, separated by septae. A nuclear medicine scan shows a non-functioning right kidney. Which is the most likely diagnosis?

a Autosomal recessive polycystic kidney disease

b Multicystic dysplastic kidney

C Multilocular cystic renal tumour

d Hydronephrosis

e Nephroblastomatosis

A

22 Answer B: Multicystic dysplastic kidney

Multicystic dysplastic kidney is the second most common cause of an abdominal mass in a neonate after hydronephrosis. The unilateral form is more common on the left and is associated with anomalies of the contralateral kidney. The bilateral form is fatal.

105
Q

33 An ultrasound of a four-year-old with an abdominal mass shows a mixed reflectivity, predominantly solid spherical mass arising from the left kidney. Further imaging in the form of a contrast-enhanced CT shows a wellcircumscribed partly cystic mass arising from the left kidney. The mass enhances less than the residual renal
parenchyma and does not contain any calcification. What is the most likely diagnosis?

a Neuroblastoma

b Nephrogenic carcinoma

c Nephroblastomatosis

d Wilms’ tumour

e Multicystic kidney

A

33 Answer D: Wilms’ tumour

Wilms’ tumour is the most common malignant abdominal neoplasm in one to four year olds. Ninety per cent of
cases present with a painless abdominal mass. The Rule of I0s applies to Wilms’ tumours:
* 10% unfavourable histology
* 10% bilateral
* 10% vascular invasion
* 10% calcification
* 10% pulmonary metastases at presentation.

106
Q

42 A six-week-old infant with a confirmed urinary tract infection presents seriously unwell with septicaemia.
Which of the following is the most appropriate investigation according to current NICE guidelines?

a DMSA during the acute infection

b DMSA four to six months after the acute infection

C MCUG with antibiotic cover during infection

d Ultrasound renal tract within six weeks

e MAG 3 renogram to exclude urinary tract obstruction

A

42 Answer B: DMSA four to six months after the acute infection

NICE guidelines refer to age group, recurrent, typical or atypical infections and the investigations vary
accordingly. A retrograde study should never be performed during an infection.

107
Q

(GU) 43 A 26-year-old male has a history of recurrent episodes of urinary tract infections and as part of a
diagnostic work-up for abdominal pain he underwent excretory urography. An IVP demonstrates early filling of a bulbous terminal right ureter which protrudes into the bladder lumen at the ureteral insertion with a surrounding radiolucent halo. There is prominence of a single proximal ureter and mild pelvicalyceal dilatation seen on the right. A MCUG demonstrates a round lucent filling defect near the trigone, which effaces with increased bladder distension. Ultrasound demonstrates a cystic mass at the right ureteral orifice at the trigone. The right kidney looks normal. During the scan, this is seen to periodically fill and empty with ureteral peristalsis. What is the most
likely diagnosis?

a Ectopic ureterocoele

b Simple ureterocoele

c TCC at the VUJ

d Ureteral oedema due to an impacted calculus

e Bladder diverticulum

A

43 Answer B: Simple ureterocoele

This is the typical cobra head' or spring onion’ appearance on IVP with a radiolucent halo formed by the ureteral wall and adjacent bladder urothelium. A simple ureterocoele is usually an incidental finding in adults; M:F = 2:3.
It may be bilateral (33%). An ectopic ureterocoele is usually seen in the upper moiety ureter of a duplex kidney (80%). Twenty per cent of ectopic ureterocoeles are seen in a single non-duplicated system and are usually associated with a small/ poorly functioning, sometimes non-visualised ipsilateral kidney (the further the ureteral
orifice is from the normal site of insertion, the more dysplastic the kidney tends to be). (c), (d) and (e) are causes of pseudoureterocoele. No protrusion of the ureter is seen into the bladder lumen.

108
Q

(GU) 49 A three-month-old male presents with a large palpable left flank mass. Ultrasound shows a solid lesion arising from the left kidney involving the renal sinus but not invading the collecting system or renal vein. What is the most likely diagnosis?

a Nephroblastomatosis

b Wilms’ tumour

C Lobar nephronia

d Mesoblastic nephroma

e Multilocular cystic nephroma

A

49 Answer D. Mesoblastic nephroma

Mesoblastic nephroma is the commonest solid renal mass in a neonate; 90% occur in the first year. It is derived
from early nephrogenic mesenchyma and can have a variable appearance on US. Distinguishing features are lack
of a well-defined cleavage plane, lack of extension into renal vein (cf. Wilms’) or pelvis (cf. multilocular cystic
nephroma). Sarcomatous transformation and metastases are rare. Treatment is nephrectomy with an excellent
prognosis.

109
Q

(GU) 56 A three-year-old female is diagnosed with juvenile onset autosomal recessive polycystic kidney disease.
Which of the following pathological features of the disease is responsible for the majority of the morbidity in this group?

a Dilated and ectatic renal collecting tubules

b Pulmonary hypoplasia

c Renal interstitial fibrosis

d Hepatic fibrosis

e Systemic arterial hypertension

A

56 Answer D: Hepatic fibrosis

ARPKD is classified into four subgroups corresponding to the age of onset: perinatal, neonatal, infantile and
juvenile. The abnormality lies with the epithelium of the collecting ducts resulting in ductal proliferation and
dilatation. The proportion of ducts involved affects the degree of renal impairment and age of onset. Those in the
juvenile group have the lowest number of ducts affected, but the degree of hepatic fibrosis is greatest in this group
and death occurs from portal hypertension.

110
Q

53 A micturating cystourethrogram performed on a child showed reflux into a tortuous ureter with a moderately dilated pelvicalyceal system, blunted forniceal angles and distinct papillary impressions. These findings represent
what grade of reflux?

a Grade I vesicoureteric reflux

b Grade II vesicoureteric reflux

C Grade III vesicoureteric reflux

d Grade IV vesicoureteric reflux

e Grade V vesicoureteric reflux

A

53 Answer D: Grade IV vesicoureteric reflux

111
Q

(GU) 61 A 15-year-old previously asymptomatic female presents with persistent thin discharge from the
umbilicus following navel piercing. Ultrasound shows a fluid filled tubular structure in the anterior abdominal
wall. What is the most likely diagnosis?

a Traumatic umbilical sinus with granulation tissue

b Patent vitello-intestinal duct

C Umbilical-urachal sinus

d Vesico-urachal diverticulum

e Umbilical-peritoneal fistula

A

61 Answer C: Umbilical-urachal sinus

The urachus is an embryological remnant of the cloaca and the allantois, and usually closes to become the median umbilical
ligament. There are four types of urachal remnant. (1) Patent urachus (continuous connection between the umbilicus and
bladder), usually symptomatic and diagnosed in the neonatal period. (2) Umbilical-urachal sinus is a blind dilatation of the
urachus at the umbilical end. A small opening into the umbilicus is usually present and trauma from the piercing has opened
it up. (3) Vesico-urachal diverticulum is a communication only with the bladder dome. (4) Urachal cyst consists of closure
at both ends with isolated dilatation usually in lower third. Meckel’s diverticulum is the remnant of the vitello-intestinal duct.

112
Q

66 A four-year-old girl presents with a three-day history of bloody diarrhoea. On examination she is pale and irritable with dry mucous membranes and a tachycardia. Blood tests reveal a microangiopathic haemolytic anaemia, thrombocytopaenia and acute renal failure. A renal ultrasound demonstrates bilateral slightly enlarged kidneys with hyperechoic cortices. What is the most likely diagnosis?

a Idiopathic thrombocytopaenic purpura

b Gram-negative sepsis

C Haemolytic uraemic syndrome

d Acute lymphoid leukaemia

e Acute glomerulonephritis

A

66 Answer C: Haemolytic uraemic syndrome

This is commonly caused by E. coli infection and is the most common cause of acute renal failure in children
requiring dialysis. The classic triad is microangiopathic haemolytic anaemia, thrombocytopaenia and acute
oliguric renal failure leading to uraemia.

113
Q

1 A two year old presents with weight loss and fatigue. On examination he has a palpable abdominal mass and
bluish nodules on the skin. He undergoes a CT of the abdomen. What are the most likely radiological findings?

a Enlarged liver containing multiple low-attenuation lesions

b Heterogeneous suprarenal mass containing calcification displacing the left kidney

c Well-circumscribed heterogeneous mass arising from the left kidney with a beak of renal tissue extending partially around the mass

d Well-circumscribed multi-septated cystic mass replacing the lower pole of the left kidney

e Solitary heterogeneous lesion within the right lobe of the liver

A

1 Answer B: Heterogeneous suprarenal mass containing calcification displacing the left kidney

The findings are those of a primary adrenal neuroblastoma with multiple metastatic skin lesions, so called
Blueberry Muffin syndrome. Thirty-six per cent of neuroblastomas arise in the adrenals and are almost always
unilateral. Common sites of metastases in decreasing order of occurrence are: bone, regional lymph nodes, orbits,
liver, brain and lung.

114
Q

4 A full-term infant born by ventouse delivery required resuscitation immediately after birth following a difficult labour. He was admitted onto the neonatal unit for observation and examination the following day revealed a palpable mass in the left lumbar region. Further tests revealed an elevated serum creatinine and haematuria.
Ultrasound showed an enlarged left kidney with loss of corticomedullary differentiation. What is the most likely diagnosis?

a Autosomal recessive polycystic kidney disease

b Renal vein thrombosis

C Wilms’ tumour

d Glomerulonephritis

e Medullary sponge kidney

A

4 Answer B: Renal vein thrombosis

Risk factors for renal vein thrombosis in neonates include advanced maternal age, diarrhoea, sepsis, birth trauma,
adrenal haemorrhage and dehydration from vomiting. It is more common on the left due to the longer left RV.

115
Q

20 A five-month-old boy is treated for a proven urinary tract infection. He does not respond well to suitable
antibiotics within 48 hours and becomes septic. Which of following would be appropriate imaging follow-up?

a Ultrasound during the acute infection, DMSA four to six months after the acute infection and MCUG

b Ultrasound during the acute infection, DMSA four to six months after the acute infection, no MCUG

c Ultrasound within six weeks of the acute infection, DMSA four to six months after the acute infection, no MCUG

d Ultrasound within six weeks of the acute infection, no DMSA or MCUG

e Ultrasound and DMSA within six weeks of the acute infection

A

20 Answer A: Ultrasound during the acute infection, DMSA four to six months after the acute infection and MCUG

An atypical infection includes septicaemia and failure to respond to appropriate antibiotics within 48 hours

116
Q

29 A two-year-old with previous urinary tract infections is investigated by ultrasound and DMSA. The ultrasound
shows possible duplex systems but normal-sized kidneys and no obvious scarring. How will the DMSA study aid the investigation?

a Differential function can be measured

b Obstruction can be demonstrated on dynamic images

c Reflux can be assessed without the need for invasive tests

d GFR can be performed based on the post DMSA blood results

e Horseshoe kidney can be excluded

A

29 Answer A: Differential function can be measured

Differential function can be measured and assessment of split function in duplex systems can be made. A horseshoe kidney may be demonstrated but it is not a primary indication for a DMSA. GFR is an entirely different
study using Tc-99m- labelled DTPA and serial blood tests. No dynamic images are obtained on DMSA and
obstruction or reflux of urinary flow is not a feature. However, the sequelae of infection, such as cortical scarring
and reduced differential function, can be accurately demonstrated.

117
Q

(GU) 35 One of the most common causes of non-prostatic urinary outflow obstruction in a male is a posterior
urethral valve. When is it usually diagnosed?

a Prenatally

b First year of life

C Early childhood

d Late childhood

e Young adult life

A

35 Answer B: First year of life

Sixty-five per cent first year, 23% 1-16 years, 8% prenatal, diagnosis by urethrogram

118
Q

41 A three-month-old girl undergoes a renal ultrasound for a urinary tract infection. This demonstrates a horseshoe
kidney. Her parents mention that she was born with very puffy hands and feet, which has now resolved. Which
of the following is the most likely underlying diagnosis?

a VACTERL (syndrome of vertebral, anal, cardiac, tracheo-oesophageal fistula/oesophageal atresia, renal and limb abnormalities)

b Turner’s syndrome

C Prader-Willi syndrome

d Noonan’s syndrome

e Edward’s syndrome

A

41 Answer B: Turner’s syndrome

Turner’s syndrome is a monosomy 45, XO. It is associated with renal ectopia and horseshoe kidney. Neonates
with Turner’s may be born with lymphoedema of the hands and feet.

119
Q

50 An 18-month-old boy is confirmed to have a large multicystic renal mass on ultrasound arising from the
kidney. On CT the mass appears to project into the renal collecting system, has enhancing septa and no nodular
components are identified. Claw-shaped adjacent normal parenchyma is seen to enhance and there is no delayed
opacification of the cystic spaces. What is the most likely diagnosis?

a Wilms’ tumour

b Cystic partially differentiated nephroblastoma

C Cystic renal cell carcinoma

d Cystic mesoblastic nephroma

e Multicystic dysplastic kidney

A

50 Answer B: Cystic partially differentiated nephroblastoma

Cystic partially differentiated nephroblastoma (CPDN) and cystic nephroma make up the two forms of
multilocular cystic renal tumour (MDCT). They originate from metanephric blastema and have identical imaging
features. CPDN is typically seen in boys aged three months to four years (90% <2 years) and blastemal cells are
present histologically. Cystic nephroma is seen typically in adult women aged 40-60 years, and no septal
blastemal cells are present. Cystic mesoblastic nephroma is rare after the age of six months (commonest neonatal
renal mass), and renal cell carcinoma is uncommon under the age of two years. Wilms’ tumour is the commonest
malignant abdominal tumour in children aged one to eight years, and the third most common cause of all renal
masses in childhood (after hydronephrosis and multicystic dysplastic kidney).

120
Q

(GU) 60 An eight-month old boy is referred for voiding cystourethrography (VCUG) to assess for reflux. He is
known to have a duplex left-sided kidney with a ureterocoele. When would the ureterocoele be most likely to be
visible?

a Early filling images

b Late filling images

c Early voiding images

d Late voiding images

e Post-void images

A

60 Answer A: Early filling images

Early filling during VCUG is essential as bladder tumours or a ureterocoele may be missed when the bladder is
full. A single AP image will suffice. During voiding steep oblique views are necessary to visualise the VUJ. AP
voiding images increase gonadal radiation dose.

121
Q

(GU) 70 A newborn finale infant had bilateral hydronephrosis detected in utero. VCUG demonstrated a bulletnosed dilatation of the posterior urethra with a trabeculated bladder and vesicoureteralreflux on the left. What is
the most likely diagnosis?

a Prune belly syndrome

b Primary megaureter

c Posterior urethral valves

d Ureterocoele

e Congenital urethral diverticulum

A

70 Answer: C: Posterior urethral valves

Posterior urethral valves are the most common cause of bilateral hydronephrosis in a male infant. There are
three types described of which type I is the most common and represents vestiges of the Wolffian duct
extending anteroinferiorly

122
Q
  1. A newborn with a history of left renal abnormalities on antenatal scans has a postnatal ultrasound which shows
    hypoechoic cysts of varying sizes with intervening abnormal renal parenchyma. There is no communication
    between the cysts. The most likely diagnosis is?

(a) Bilateral hydronephrosis

(b) Autosomal dominant polycystic kidney disease

(c) Wilms’ tumour

(d) Autosomal recessive polycystic kidney disease

(e) Multicystic dysplastic kidney

A
  1. (e) Multicystic dysplastic kidney

The sonographic appearances of this condition are typical, showing noncommunicating cysts of varying sizes
separated by hyperechoic renal parenchyma (differentiating it from hydronephrosis). Autosomal dominant
polycystic kidney disease is seen in adults. Autosomal recessive polycystic kidney disease shows bilateral gross
renal enlargement with hyperechoic kidney involvement and usually no cysts are seen on ultrasound.

123
Q
  1. A 14-month-old child presents with abdominal pain and vomiting. Ultrasound shows a dilated left renal pelvis
    and calyces. CT shows a massively dilated renal pelvis with tapering at the inferior margin. The ureters are not
    dilated. There is a delayed nephrogram in the left kidney with no contrast excretion in the left kidney and normal
    on the right. A MAG-3 scan shows a split renal function of 80% on the right and 20% on the left. The most likely
    diagnosis is?

(a) Multicystic dysplastic kidney

(b) Congenital megacalyces

(c) Primary megaureter

(d) Congenital pelvic–ureteric junction obstruction

(e) Autosomal dominant polycystic kidney disease

A
  1. (d) Congenital pelvic–ureteric junction obstruction
    Findings of the MAG-3 scan along with typical imaging demonstrating obstruction at the pelviureteric junction.
124
Q

(GU) 23. A child was diagnosed with prune-belly syndrome. Which of the following is most likely to be a feature
associated with this condition?

(a) Polyhydramnios in mother

(b) Usually seen in women

(c) Normal bladder capacity

(d) Vesicoureteral reflux usually present

(e) Normal bladder neck

A
  1. (d) Vesicoureteral reflux usually present

With an abnormal urinary tract, there may be failure of fetal micturition and thus oligohydramnios. This is seen exclusively in men and reflux is present in the majority of cases. The bladder neck is typically wide with a tapering dilatation of the posterior urethra.

125
Q
  1. A newborn infant was diagnosed with bilateral hydronephrosis in utero on antenatal ultrasound. A micturating
    cystourethrogram shows a ‘bullet’ shaped dilatation of the posterior urethra and a thin urethral calibre, a thickwalled and trabeculated urinary bladder, with severe vesicoureteral reflux on the left side. What is the most likely
    diagnosis?

(a) Bilateral pelvic–ureteric junction obstruction

(b) Posterior urethral valves

(c) Vesicoureteral obstruction

(d) Primary megaureter

(e) Megacystis–microcolon–intestinal hypoperistalsis syndrome

A
  1. (b) Posterior urethral valves

There are congenital folds of mucous membrane located in the posterior urethra. The findings on micturating cystourethrogram are characteristic with fusiform distension and elongation of posterior urethra, vesicoureteral reflux (usually to the left), diminution of urethral calibre and hypertrophy of bladder neck with trabeculations.

126
Q
  1. A 2-year-old child presents with abdominal swelling. CT shows a large mass pushing the right kidney and
    encasing the inferior vena cava and aorta. There are speckled calcifications seen in the lesion. What is the most
    likely diagnosis?

(a) Neuroblastoma

(b) Lymphoma

(c) Wilms’ tumour

(d) Multicystic kidney

(e) Mesoblastic nephroma

A
  1. (a) Neuroblastoma

A childhood suprarenal mass with calcification, crossing the midline to encase the inferior vena cava and aorta is
almost certainly a neuroblastoma.

127
Q

(GU) 31. A 4-year-old child was referred for a palpable abdominal mass and abdominal pain. Ultrasound shows
a large heterogenous mass in the abdomen. Contrast enhanced CT demonstrates a large heterogenous and necrotic
mass arising from the right kidney, extending across the midline and displacing the aorta and inferior vena cava.
There are calcifcations within the lesion. What is the most likely diagnosis?

(a) Neuroblastoma

(b) Nephroblastoma

(c) Renal cell carcinoma

(d) Oncocytoma

(e) Hepatoblastoma

A
  1. (b) Nephroblastoma

Also called Wilms’ tumour, this lesion typically presents as a large mass, crossing the midline and commonly
displacing the large vessels (in contrast to neuroblastoma, where the mass encases the vessels). This is the most
common neoplasm in children between 1 and 8 years of age.

128
Q
  1. A toddler presents with urinary retention and abdominal distension. CT shows a large pelvic mass with
    calcifications. T2 and STIR images on MRI demonstrate a large, predominantly solid, mixed signal intensity mass
    in the presacral region, which extends in between the sacral segments, encasing the sacrum. The bladder and
    rectum are displaced anteriorly but not invaded. What is the most likely diagnosis?

(a) Ovarian teratoma

(b) Neuroblastoma

(c) Sacrococcygeal germ cell tumour

(d) Anterior meningocele

(e) Duplication rectum

A
  1. (c) Sacrococcygeal germ cell tumour

These are relatively rare tumours and can be benign or malignant. Calcification is seen on CT in more than half
of cases, more frequently in benign lesions. Direct invasion of surrounding structures suggest malignancy.
Ovarian teratoma, neuroblastoma and gastrointestinal duplication cysts are all rare and do not (usually) extend
around the sacrum. Meningoceles are cystic structures.

129
Q

(GU) 43. A 20-week routine anatomy scan of a pregnant woman shows oligohydramnios. The kidneys were
hyperechoic and enlarged with a renal: abdominal circumference ratio of 0.35. Te urinary bladder was empty.
What is likely diagnosis?

(a) Autosomal recessive polycystic kidney disease

(b) Autosomal dominant polycystic kidney disease

(c) Posterior urethral valves

(d) von Hippel–Lindau disease

(e) Nephroblastomatosis

A
  1. (a) Autosomal recessive polycystic kidney disease

This condition can be diagnosed as early as 17–18 weeks on obstetric ultrasound. The kidneys may be massively enlarged measuring up to 10–20 times normal size with an enlarged renal: abdominal circumference ratio of >
0.30. The renal parenchyma is hyperechoic and there may be oligohydramnios. There is an association with
congenital hepatic fbrosis.

130
Q

(GU) 44. A 3-month-old baby presents with an abdominal lump. Ultrasound shows a large solid mass arising
from the right kidney with focal hypoechoic areas. Contrastenhanced CT shows a solid right renal mass, involving
the renal sinus and multiple small areas of necrosis. There is no invasion of the renal vein or the collecting system.
No metastatic deposits are seen. What is the most likely diagnosis?

(a) Wilms’ tumour

(b) Mesoblastic nephroma

(c) Nephroblastomatosis

(d) Renal metastases

(e) Lymphoma of kidney

A
  1. (b) Mesoblastic nephroma

This is a hamartoma and is the most common solid neoplasm in neonates. It typically involves the renal sinus and there is no invasion of the veins (differentiating it from Wilms’ tumour) or the collecting system.

131
Q

(GU) 45. A 10-year-old girl presents with urinary tract infection. Ultrasound and micturating cystourethrogram
demonstrates a left-sided vesicoureteral reflux with reflux to the pelvicalyceal system without calyceal dilatation
or blunting. What is the most likely grade of the vesicoureteral reflux?

(a) Grade I

(b) Grade II

(c) Grade III

(d) Grade IV

(e) Grade V

A
  1. (b) Grade II

Grade I is reflux only in the ureter and not into the pelvicalyceal system.
Grade III is reflux into the pelvicalyceal system with mild dilatation of the ureter and pelvicalyceal system.
Grade IV and V are more severe dilatations and tortuosity of the ureter and pelvicalyceal system.

132
Q
  1. A 12-year-old boy was diagnosed with prune belly syndrome. Which of the following statements is false?

(a) It is seen exclusively in men

(b) Bilateral undescended testis

(c) Abdominal wall deficiency

(d) Non-obstructed and dilated ureters

(e) Non-hereditary multisystem disorder

A
  1. (e) Non-hereditary multisystem disorder

Prune belly syndrome is a congenital non-hereditary multisystem disorder, almost exclusive to men (M:F = 19:1).
A triad of abdominal wall deficiency, non-obstructed dilated redundant ureters and bilateral undescended testes
is seen

133
Q

(GU) 1. Which of the following are correct of multicystic dysplastic kidneys? (T/F)

(a) Antenatal ultrasound reveals a paraspinal mass with multiple communicating cysts.

(b) Approximately 30% of contralateral kidneys are abnormal.

(c) It is more common in infants of diabetic mothers

(d) Upto 20% of cases have residual renal function in the affected kidney on scintigraphy.

(e) Nephrectomy is usually required, as it cannot be differentiated from cystic Wilms tumour.

A

Answers:

(a) Not correct
(b) Correct
(c) Correct
(d) Correct
(e) Not correct

Explanation:

Antenatal ultrasound shows multiple non-communicating cysts of variable size. Serial ultrasound scans of
multicystic dysplastic kidney show renal involution. Nephrectomy is required if it fails to involute or if there is uncontrolled hypertension. Multicystic dysplastic kidney is the second most common cause of neonatal abdominal
mass (first is hydronephrosis).

134
Q

(GU) 5. Concerning ureterocoeles, which of the following are correct? (T/F)

(a) About 80% of ectopic ureterocoeles occur in duplicated systems.

(b) In children, a pseudoureterocoele most commonly results from an impacted stone.

(c) They can be indistinguishable from bladder diverticula during voiding.

(d) A simple ureterocoele never occurs in an ectopic ureter.

(e) A single simple ureterocoele can cause bladder outlet obstruction.

A

Answers:

(a) Correct
(b) Correct
(c) Correct
(d) Correct
(e) Correct

Explanation:

Ureterocoele is a congenital dilatation of distal most portion of ureter. In minority of cases it is an isolated finding and seen in adults. On IVP it shows ‘cobra head’ sign. Complications include obstructive cystic renal dysplasia.

135
Q

(GU) 7. In prune belly syndrome, which of the following are correct? (T/F)

(a) The condition is more common in females.

(b) The inheritance is autosomal recessive.

(c) It is characterised by markedly distended and obstructed ureters.

(d) There is an association with Hirschsprung’s disease.

(e) Death occurs within 1 year of birth in the majority of cases.

A

Answers:

(a) Not correct
(b) Not correct
(c) Not correct
(d) Correct
(e) Not correct

Explanation:

Prune belly syndrome is a sporadic nonhereditary condition, exclusively affecting males. It is characterized by
triad of absent anterior abdominal wall muscles, non-obstructed megaureters and cryptorchidism (due to bladder
distension). In 20% cases death occurs in first month of life and another 30% within 2 yrs of life. Mild cases
survive into adulthood.

136
Q
  1. Regarding infantile polycystic kidney disease, which of the following are correct? (T/F)

(a) The most common age of presentation is between 2 and 5 years of age.

(b) It is inherited as an autosomal dominant condition.

(c) Severe infantile polycystic disease is associated with severe hepatic fibrosis.

(d) Is typically reveals a striated nephrogram on the delayed excretory urogram.

(e) Infantile polycystic kidneys are echopoor on ultrasound.

A

Answers:

(a) Not correct
(b) Not correct
(c) Not correct
(d) Correct
(e) Not correct

Explanation:

Infantile polycystic kidney disease is inherited in an autosomal recessive pattern with antenatal form being the
most common type. This type presents in utero and progresses to renal failure and pulmonary hypoplasia (Potter
sequence) with majority of patients dying within 24 hrs of life. The milder forms are neonatal, infantile and
juvenile presenting in the first few years of life. They are associated with more severe hepatic fibrosis and less
severity of renal disease. The affected kidneys are replaced my multiple small elongated cysts representing dilated
tubules and collecting ducts. The cysts are too small to be delineated on ultrasound, thus producing an echogenic
pattern due to multiple interfaces. There is poor corticomedullary differentiation. Adult type polycystic disease
shows autosomal dominant inheritance with defects on chromosome 16 and 4.

137
Q
  1. Which of the following statements are correct? (T/F)

(a) Vesico-ureteric reflux usually occurs into the lower pole moiety of a complete ureteral duplication

(b) Ectopic ureterocoeles in duplex kidneys are more common in boy than girls.

(c) An ectopic ureteral insertion may present with daytime incontinence in a girl.

(d) Horseshoe kidneys are associated with a higher incidence of duplicated kidneys.

(e) Ectopic ureteral insertion in boys in always supra-sphincteric.

A

Answers:

(a) Correct
(b) Not correct
(c) Correct
(d) Correct
(e) Correct

Explanation:

Ureterocoeles which are related to the upper pole moiety of a duplex kidney occur 8 times more frequently in
girls than boys. Ureterocoeles related to non-duplicated system show an equal gender incidence.

138
Q

(GU) 19. Which of the following statements are correct? (T/F)

(a) Haematocolpos refers to a uterine cavity distended with blood.

(b) After puberty, the most common cause of haematocolpos is vaginal atresia.

(c) A fluid collection within the endometrial cavity of a neonate is abnormal.

(d) Congenital haematometrocolpos is usually associated with other anomalies.

(e) Haematometra is a cause of ureteric obstruction.

A

Answers:

(a) Not correct.
(b) Not correct
(c) Not correct
(d) Correct
(e) Correct

Explanation:

Hematocolpos refers to vagina distended with blood and hematometra refers to uterine cavity distended with blood. An imperforate hymen is the most common cause of hematocolpos. Approximately 25% of neonates have
a fluid collection within the endometrial cavity, thus a normal finding.

139
Q
  1. The following features favour a diagnosis of Wilm’s tumor rather than neuroblastoma: (T/F)

(a) Presentation before 1 year of age.

(b) Calcification

(c) Lung metastases

(d) Claw sign

(e) The tumor displaces rather than engulfs the major vessels

A

Answers:

(a) Not correct
(b) Not correct
(c) Correct
(d) Correct
(e) Correct

Explanation:

Peak age of Wilm’s tumor is 3 years and neuroblastoma is most common in children below 2 years. Stippled type
of calcification is seen in 85% of cases of neuroblastoma. Calcification is uncommon in Wilm’s and is seen as
curvilinear or amorphous type in 15% cases.

140
Q
  1. Which of the following are correct regarding renal masses in neonates: (T/F)

(a) Wilm’s tumor is the most common cause of an abdominal mass in neonates.

(b) Mesoblastic nephroma is the most common fetal renal reoplasm.

(c) Mesoblastic nephroma is easily distinguished from Wilm’s tumor on cross-sectional imaging

(d) Extension into the renal vein is a feature of mesoblastic nephroma.

(e) Mesoblastic nephroma is associated with oligohydramnios.

A

Answers:

(a) Not correct
(b) Correct
(c) Not correct
(d) Not correct
(e) Not correct

Explanation:

Hydronephrotic kidney is the most common abdominal mass in neonates. Mesoblastic nephroma is associated
with polyhydramnios and thus premature labor. It cannot be distinguished from Wilm’s tumor on cross-sectional
imaging. It is benign and does not invade venous structures, hence differentiating it from Wilm’s tumor.

141
Q

@#e (GU) 34. Which of the following are correct regarding multicystic dysplastic kidney: (T/F)

(a) Is the second commonest cause of a neonatal abdominal mass.

(b) Is associated with PUJ obstruction.

(c) Is usually unilateral.

(d) The renal cysts communicate.

(e) Intervening normal renal parenchyma is present.

A

Answers:

(a) Correct
(b) Correct
(c) Correct
(d) Not correct
(e) Not correct

Explanation:

Multicystic dysplastic kidney shows non-communicating cysts with no normal renal parenchyma.